You are on page 1of 259

Complete Practice Question Set

Aryabhatta National Maths Competition - 2023

Class - 10
TIME AND WORK

1. If A can do a piece of work in n days, then A's 1 day work = 1/n.

2. If A's 1 day work = 1/n, then A can finish the work in n days.

3. If A is twice as good a workmen as B, then

o Ratio of work done by A and B in the same time = 2:1

o Ratio of time taken by A and B in doing the same work = 1:2

Q1. If Ganpat completes work in 6 days and Prashant can complete the same work
in 2 days, in how many days both will complete it?

A. days

B. days

C. days

D. days

Answer: C

Explanation:

Ganpat’s 1 day’s work = , Prashant’s 1 day’s work =


(Ganpat+ Prashant)’s 1 day’s work = ( + )=

Both Ganpat and Prashant will complete the work in = days

Q2. P and Q together can complete work in 14 days and Q alone in 18 days. In how
many days can P alone complete the work?

A. days

B. days

C.59days

D. days

Answer: B

Explanation:

(P + Q)’s 1 day’s work = , Q’s 1 day’s work =

P’s 1 day’s work = ( - )=


Hence, P alone can complete the work in 63 days.

Q3. R can do work in 5 days, S in 7 days and T in 14 days. Find the time taken by R, S
and T to do the work together.

A. days
B. days

C. days

D. days

Answer: B

Explanation:

R’s 1 day’s work = , S’s 1 day’s work = , T’s 1 day’s work =

(R + S + T) ’s 1 day’s work = ( + )=

Hence, R, S and t together can do the work in

Q4. Rahul can do work in 9 days of 6 hours each and Rohan can do it in 8 days of 4

hours each. How long will they take to do it, working together 5 hours a day?

A. 3.46 days

B. 4.5 days

C. 2.90days

D. 1.49days

Answer: A
Explanation:

Rahul can complete the work in (9 × 6) = 54 hours

Rohan can complete the work in (8 × 4) = 32 hours.

∴ Rahul’s 1 hour’s work = 1/54 and Rohan’s 1 hour’s work = 1/32

(Rahul +Rohan)’s 1 hour’s work = =

∴ Both will finish the work in = hrs.

Number of days of 5 hrs each = ( x ) = 3.46

Q5. A and B can complete the work in 12 days; B and C can do it in 15 days; A and C
can do it in 20 days. In how many days will A, B and C finish it, working together?

A. 10 days

B. 8 days

C. 11days

D. 15days

Answer: A

Explanation:

(A + B)’s 1 day’s work = , (B + C)’s 1 day’s work =

(A + C)’s 1 day’s work =


Adding, we get: 2 (A + B + C)’s 1 day’s work =

∴ (A + B + C)’s 1 day’s work =

Thus, A, B and C together can finish the work in 10 days.

Q6. Sahil is four times the best workman than Dinesh and both can finish work in 12
days. In how many days will Sahil alone complete the work ?

A. 14

B. 12

C. 10

D. 15

Answer: D

Explanation:

(Sahil’s 1 day’s work) : (Dinesh’s 1 day’s work) = 4 : 1

(Sahil + Dinesh)’s 1 day’s work =

Divide 1/12 in the ratio 4 : 1.

∴ Sahil’s 1 day’s work = ( * )=


Hence, Sahil alone can finish the work in 15 days.

Q7. Car is 44% more efficient than a Bike. Bike takes 9 days to complete. How many
days does a car alone take to complete a journey ?
A.

B.

C.

D.

Answer: D

Explanation:

Ratio of times taken by car and bike = 144 : 100 = 36 : 25

Suppose B alone takes x days to do the job.

Then, 36 : 25 :: 9 : x ⇒ 36x = 25 × 9 ⇒ x = days

Q8. E can do a piece of work in 48 days. He works for 12 days and then F alone
finishes the remaining work in 36 days. In how much time will both together
complete the work ?

A. 24 days

B. 20 days

C. 18days

D. 16days

Answer: A

Explanation:
Work done by E in 12 days = ( *12) = .

Remaining work = (1 - ) =

Now, 3/4 work is done by F in 36 days.

Whole work will be done by F in ( *36) = 48 days

∴ E’s 1 day’s work = andF’s 1 day’s work =

∴ (E+ F)’s 1 day’s work = + )=


Hence, both will finish the work in 24 days

Q9. Ram does work in 8 days and Sham in 24 days. They both worked together but 4
days before the completion of the work, Ram leaves due to some health issue. In
how many days the work will get completed?

A.18days

B. 6 days

C. 11days

D. 8days

Answer: A

Explanation:

(ram + sham)’s 1 day’s work = ( + )=


Ram & Sham can complete work in 6 days

Since 4 days before the job is completed (I,e, 2 days post starting), Ram left

2/3 of work is pending.

Sham can complete work in 24 days

∴ Extra time taken by Sham is = 24*2/3= 16 days

Hence, total time taken = (2 + 16) days = 18 days.

Q10. A can complete a work in 7 days while B can complete the same work in 14
days. If they work together and complete it, the portion of the work done by A is

A. days

B. days

C. days

D. days

Answer: A

Explanation:

(A + B)’s 1 day’s work = ( + )=

Both A and B together can complete the work in days.


Part of the work done by A = ( x )=

Q11. Vedu and Yash can sow a field in 6 days. Vedu alone can sow the field in 8 days.
In how many days will Yash alone sow the field?

A.24days

B.30 days

C.12 days

D.18 days

Answer: A

Explanation:

Yash 1 day’s work = ( - ) =

Yash alone can sow the field in 24 days

Q12. M and N can do a piece of work in 15 days. N and O together can do it in 20


days. If M is twice as good a workman as O, find in what time N alone can do it?

A.30 days

B.35 days

C. 27 days

D.25days

Answer: A

Explanation:
M’s 1 day’s work = O’s 2 days’ work.

(M + N)’s 1 day’s work = (N’s 1 day’s work) + (O’s 2 days’ work)

⇒ (N’s 1 day’s work) + (O’s 2 days’ work) = ……………(i)

But (N’s 1 day’s work) + (O’s 1 day’s work) = ……………(ii)

Subtracting (ii) from (i), we get: O’s 1 day’s work = ( - )=

N’s 1 day’s work = ( - )=

Hence, N alone can finish the work in 30 days.

Q13. 55 women can complete a work in 15 days. five days after they started working,
25 more women joined them. How many days will they now take to complete the
remaining work ?

A. 6 days

B. 7 days

C. 8 days

D. 9 days

Answer: B

Explanation:

(55 × 15) women can complete the work in 1 day.

⇒ ∴ 1 woman’s 1 day’s work = . 55 women's 1 days' work = =


55 women’s 5 days’ work = ( *5)= Remaining work = ( - )=

80 women’s 1 day’s work = =

Now, work is done by them in 1 day.

work is done by them in ( * ) = 6.87 daysi.e. 7 days

Q14. 8 students begin to work together on a project but after some days 3 students
leave. As a result, the project which could have been completed in 30 days is
completed in 40 days. How many days after the commencement of the work did the
3 students leave?

A. 16 days

B. 14 days

C. 20 days

D. 11 days

Answer: B

Explanation:

8 students can complete the work in 30 days.

∴ 1 students 1 day’s work = .


Suppose 3 students left after x days.
Then, x x + x x (40-x) = 1

⇒ + (40-x) = 1

⇒ + - x=1

⇒ =

⇒ x = 13.33

Hence, 2 student left 14 days after the commencement of the work

Q15. There are two buckets. The first bucket filled in 8 minutes and the second alone
filled in 5 minutes. If buckets are filled at a constant rate, how long does it take both
the buckets together to be filled ?

A. days

B. days

C. days

D. days

Answer: A

Explanation:

1 minute’s work of both the buckets =


So, both the punctures will make the type flat in =

Q16. R can finish a work in 21 days and T can do the same work in one third the time
taken by R. Then, working together, what part of the same work can they finish in a
day ?

A.

B.

C.

D.

Answer: A

Explanation:

R’s 1 day’s work = andT’s 1 day’s work =

∴ (R + T)’s 1 day’s work = =

Q17. A farmer and his child together can do farming in 60 days. Their speeds in
farming are in the ratio of 10 : 6. How many days will a child take to complete the
work if engaged alone?

A. 96 days

B. 160 days
C. 85 days

D. 130 days

Answer: B

Explanation:

Ratio of farming speeds of farmer and child = 10 : 6.

Ratio of times taken by man and boy = 6 : 10.

Suppose the man takes 6x days while the boy takes 10x days to complete the work
alone.

Then, + =

⇒ =

⇒ = 16

Hence, time taken by the boy to complete the work alone

= (10 × 16) days = 160 days.

Q18. Sita can eat 28 golgappas in half an hour. Her sister Geeta needs two hours to
eat the same number. How much time will they take to eat 28 golgappas together?

A. 32 min

B. 24 min

C. 20 min

D. 36 min
Answer: B

Explanation:

Number of golgappas eaten by Sita in 1 minute =

Number of golgappas eaten by Geeta in 1 minute =

Number of golgappas eaten by Rosa and Lila together in

1 minute = + =

⇒ Required time = / = min = 24 min.

Q19. P, Q and R can separately do work in 16, 20 and 25 days respectively. They
started to work together but R left after 4 days. The remaining work will be finished
in.

A. 4 days

B. 6 days

C. 2 days

D. 5 days

Answer: A

Explanation:

(P + Q + R )’s 1 day's work = =


(P + Q + R)’s 4 days’ work = =

Remaining work = (1- )=

(P + Q)’s 1 day’s work = =

Now, 9/80 work is done by P and Q in 1 day.

39/100 work will be done by P and Q in =3.46 days

= 4 days

Q20. Six men can complete a work in 10 days. They started the work and after 6
days, three men left. In how many days will the work be completed by the remaining
men ?

A. 8 days

B. 10 days

C. 6 days

D. 12 days

Answer: A

Explanation:

(6 × 10) men can complete the work in 1 day.

1 man’s 1 day’s work =


6 men’s 6 days’ work = days = *6 =

Remaining work = 1- =

3 men’s 1 day’s work = *3 =

work is done by them in 1 day.

work is done by them in = .

Q21. C and D together can do a job in 6 days; D and E can do it in 9 days; C and E in
days. The number of days required for C to do the job alone is ?

A. 6 days

B. 8 days

C. 9 days

D. 10 days

Answer: A

Explanation:

(C + D)’s 1 day’s work =

(D + E)’s 1 day’s work =

(C + E)’s 1 day’s work =


Adding, we get: 2 (C + D + E)’s 1 day’s work

= =

(C + D + E)’s 1 day’s work =

So, C’s 1 day’s work = =

C alone can do the work in 6 days.

Q22. Thirty girls can do a job in twelve days. twelve boys can complete the same job
in ten days. What is the ratio between the capacity of a boy and girl ?

A. 3: 1

B. 1 : 6

C. 2 : 5

D.1 : 3

Answer: A

Explanation:

(30 × 12) girls can complete the work in 1 day

1 girl’s 1 day’s work =

(12 × 10) boys can complete the work in 1 day.

1 boy’s 1 day’s work =

So, required ratio = : = 3 : 1.


Q23. Eight boys can complete a job in sixteen days. They started the work and after
4 days, three boys left. In how many days will the job be completed by the
remaining boys ?

A. days

B. days

C. days

D. days

Answer: A

Explanation:

(8 × 16) boys can complete the work in 1 day

1 boy’s 1 day’s work =

8 boy’s 4 days’ work = =

Remaining work = 1- =

5 boy’s 1 day’s work = *5 =

5/128 work is done by them in 1 day.

3/4 work is done by them in = = days


Q24. 15 boys can do a job in 32 days. How many days are needed to complete the
work, if 12 boys do this work?

A. 40

B. 32

C. 26

D. 25

Answer: A

Explanation:

15 boys can do a piece of work in 32 days

⇒ B1 = 15 and D1 = 32

12 boys can do this work in D2 days

⇒ B2 = 12

B1 D1 = B2 D2

⇒ 15 × 32 = 12 × D2

⇒ D2 = = 40 days

Q25. Priya can do work in 18 days. Tejal is 55% more efficient than Priya. The
number of days Tejal will take to do the same piece of work, is?

A. days

B. days
C. days

D. days

Answer: A

Explanation:

Ratio of times taken by Tejal and Priya = 155 : 100 = 31 : 20

Suppose Tejal alone takes x days to do the job.

Then, 31 : 20 :: 18 : x ⇒ 31x = 20 × 18 ⇒ x = days

Q 26 - A can do a bit of work in 8 days, which B alone can do in 10 days in how


long . In how long both cooperating can do it?

A - 40/9 days

B - 41/9 days

C - 42/9 days

D - 43/9 days

Answer - A

Explanation

A's 1 day work= 1/8, B`s 1 day work = 1/10


∴ (A+B) 1 day work = (1/8+1/10) = 9/40
Both cooperating can complete it in 40/9 days.

Q 27 - A and B together can dive a trench in 12 days, which an alone can dive
in 30 days. In how long B alone can burrow it?

A - 18 days

B - 19 days

C - 20 days

D - 21 days

Answer - C

Explanation

(A+B)'s 1 day work = 1/12, A's 1 day work =1/30


∴ B's 1 day work = (1/12-1/30) = 3/60 = 1/20
Henceforth, B alone can dive the trench in 20 days.

Q 28 - A can do a bit of work in 25 days which B can complete in 20 days. Both


together labor for 5 days and afterward A leaves off. How long will B take to
complete the remaining work?

A - 7 days

B - 8 days
C - 9 days

D - 11 days

Answer - D

Explanation

(A+B)'s 5 days work = 5(1/25+1/20) = (5*9/100) = 9/20


Remaining work = (1-9/20) = 11/20
1/20 work is finished by B in 1 day
11/20 work is finished by B in (1*20*11/20) = 11 days

Q 29 - A and B can do a bit of work in 12 days. B and C can do it in 15 days


while C and A can do it in 20 days. In how long will they complete it
cooperating? Additionally, in how long can A alone do it?

A - 10 days, 30 days.

B - 15 days, 20 days.

C - 20 days, 40 days.

D - 10 days, 50 days.

Answer - A

Explanation

(A+B)'s 1 day work = 1/12,


(B+C)'s 1 day work = 1/15,
(C+A)'S 1 day work = 1/20
Including: 2(A+B+C)'s 1 day work = (1/12+ 1/15+ 1/20)= 12/60 = 1/5
∴ (A+B+C) `s 1 day work = (1/2 *1/5) = 1/10
∴ working together they can complete the work in 10 days.
A's 1 day work = (1/10-1/15) = 1/30, B`s 1 day work = (1/10-1/20) = 1/20
C's 1 day work = (1/10-1/12) = 1/60
∴ A alone can take the necessary steps in 30 days.

Q 30 - A can fabricate a divider in 30 days , while B alone can assemble it in 40


days, If they construct it together and get an installment of RS. 7000, what B's
offer?

A - 2000

B - 3000

C - 4000

D - 6500

Answer - B

Explanation

A's 1 days work = 1/30,


B's 1 day work = 1/40,
Proportion of their shares = 1/30:1/40 = 4:3
B's offer = (7000*3/7) = Rs. 3000

Q 31- A can do a bit of work in 10 days while B alone can do it in 15 days. They
cooperate for 5 days and whatever remains of the work is finished by C in 2
days. On the off chance that they get Rs. 4500 for the entire work, by what
means if they partition the cash?

A - Rs 1250, Rs 1200, Rs 550

B - Rs 2250, Rs 1500, Rs 750

C - Rs 1050, Rs 1000, Rs 500

D - Rs 650, Rs 700, Rs 500

Answer - B

Explanation

(A+B)'s 5 days work = 5(1/10+ 1/15)= (5* 1/6)= 5/6


Remaining work = (1-5/6) = 1/6
C's 2 days work = 1/6
(A's 5 day work): (B's 5 day work): (C's 2 days work)
= 5/10: 5/15: 1/6
= 15: 10:5 = 3:2:1
A's offer = (4500*3/6) = Rs. 2250
B's offer = (4500*2/6) = Rs. 1500
C's share= (4500*1/6) = Rs. 750
Q 32 - A, B and C can do a piece of work in 24 days, 30 days and 40 days
respectively. They began the work together but C left 4 days before the
completion of the work. In how many days was the work completed?

A - 11 Days

B - 12 Days

C - 13 Days

D - 14 Days

Answer: A) 11 days

Explanation

One day's work of A, B and C = (1/24 + 1/30 + 1/40) = 1/10.

C leaves 4 days before completion of the work, which means only A and B work
during the last 4 days.

Work done by A and B together in the last 4 days = 4 (1/24 + 1/30) = 3/10.

Remaining Work = 7/10, which was done by A,B and C in the initial number of
days.

Number of days required for this initial work = 7 days.

Thus, the total numbers of days required = 4 + 7 = 11 days.


Q 33 - P can complete a work in 12 days working 8 hours a day.Q can
complete the same work in 8 days working 10 hours a day. If both p and Q
work together,working 8 hours a day,in how many days can they complete the
work?

A - 60/11

B - 61/11

C - 71/11

D - 72/11

Answer: A) 60/11

Explanation
P can complete the work in (12 x 8) hrs = 96 hrs
Q can complete the work in (8 x 10) hrs=80 hrs
Therefore, P's 1 hour work=1/96 and Q's 1 hour work= 1/80
(P+Q)'s 1 hour's work =(1/96) + (1/80) = 11/480. So both P and Q will finish the
work in 480/11 hrs

Therefore, Number of days of 8 hours each = (480/11) x (1/8) = 60/11

Q 34 - A and B can do a piece of work in 30 days , while B and C can do the


same work in 24 days and C and A in 20 days . They all work together for 10
days when B and C leave. How many days more will A take to finish the work?
A - 18 Days

B - 24 Days

C - 30 Days

D - 36 Days

Answer: A) 18 days

Explanation

2(A+B+C)'s 1 day work = 1/30 + 1/24 + 1/20 = 1/8

=>(A+B+C)'s 1 day's work= 1/16

work done by A,B and C in 10 days=10/16 = 5/8

Remaining work= 3/8

A's 1 day's work= (1 / 16−1 / 24)=1 / 48

Now, 1/48 work is done by A in 1 day.

So, 3/8 work will be done by A in =48 x (3/8) = 18 days

Q 35 - An air conditioner can cool the hall in 40 miutes while another takes 45
minutes to cool under similar conditions. If both air conditioners are switched
on at same instance then how long will it take to cool the room approximately
?
A -18 minutes

B - 19 minutes

C - 22 minutes

D - 24 minutes

Answer: C) 22 minutes

Explanation

Let the two conditioners be A and B

'A' cools at 40min

'B' at 45min

Together = (a x b)/(a + b)

= (45 x 40)/(45 + 40)

= 45 x 40/85

= 21.1764

= 22 min (approx).
Q 36 - A works twice as fast as B.If B can complete a work in 18 days
independently,the number of days in which A and B can together finish the
work is:

A - 4 days

B - 6 days

C - 8 days

D - 10 days

Answer: B) 6 days

Explanation

Ratio of rates of working of A and B =2:1. So, ratio of times taken =1:2

Therefore, A's 1 day's work=1/9

B's 1 day's work=1/18

(A+B)'s 1 day's work= 1/9 + 1/18 = 1/6

so, A and B together can finish the work in 6 days

Q 37 - A can do a certain work in the same time in which B and C together can
do it.If A and B together could do it in 20 days and C alone in 60 days ,then B
alone could do it in:

A - 20 days
B - 40 days

C - 50 days

D - 60 days

Answer: D) 60 days

Explanation

(A+B)'s 1 day's work=1/20

C's 1 day work=1/60

(A+B+C)'s 1 day's work= 1/20 + 1/60 = 1/15

Also A's 1 day's work =(B+C)'s 1 day's work

Therefore, we get: 2 x (A's 1 day 's work)=1/15

=>A's 1 day's work=1/30

Therefore, B's 1 day's work= 1/20 - 1/30 = 1/60

So, B alone could do the work in 60 days.

Q 38 - A is thrice efficient as B and C is twice as efficient as B. what is the ratio


of number of days taken by A,B and C, when they work individually?

A - 2:6:3
B - 2:3:6

C -1:2:3

D - 3:1:2

Answer: A) 2:6:3

Explanation
A : B : C

Ratio of efficiency 3 : 1 : 2

Ratio of No.of days 1/3 : 1/1 : 1/2

or 2 : 6 : 3

Hence A is correct.

Q 39 - 4 men can repair a road in 7 hours. How many men are required to
repair the road in 2 hours ?

A - 17 men

B - 14 men

C - 13 men

D - 16 men

Answer: B) 14 men
Explanation

M x T / W = Constant
where, M= Men (no. of men)
T= Time taken
W= Work load
So, here we apply
M1 x T1/ W1 = M2 x T2 / W2
Given that, M1 = 4 men, T1 = 7 hours ; T2 = 2 hours, we have to find M2 =?
Note that here, W1 = W2 = 1 road, ie. equal work load.
Clearly, substituting in the above equation we get, M2 = 14 men.

Q 40 - A,B,C together can do a piece of work in 10 days.All the three started


workingat it together and after 4 days,A left.Then,B and C together completed
the work in 10 more days.In how many days can complete a work alone ?

A - 25

B - 24

C - 23

D - 21

Answer: A) 25
Explanation

(A+B+C) do 1 work in 10 days.

So (A+B+C)'s 1 day work=1/10 and as they work together for 4 days so


workdone by them in 4 days=4/10=2/5

Remaining work=1-2/5=3/5

(B+C) take 10 more days to complete 3/5 work. So( B+C)'s 1 day work=3/50

Now A'S 1 day work=(A+B+C)'s 1 day work - (B+C)'s 1 day work=1/10-


3/50=1/25

A does 1/25 work in in 1 day

Therefore 1 work in 25 days.

Q 41 - (x-2) men can do a piece of work in x days and (x+7) men can do 75% of
the same work in (x-10)days. Then in how many days can (x+10) men finish
the work?

A - 27 Days

B - 12 Days

C - 25 Days

D - 18 Days
Answer: B) 12 days

Explanation

34×(x−2)x=(x+7)(x−10)34×(x-2)x=(x+7)(x-10)

⇒x2−6x−280 =0⇒x2-6x-280 =0

=> x= 20 and x=-14

so, the acceptable values is x=20

Therefore, Total work =(x-2)x = 18 x 20 =360 unit

Now 360 = 30 x k

=> k=12 days

Q 42 - 3 men, 4 women and 6 children can complete a work in 7 days. A


woman does double the work a man does and a child does half the work a
man does. How many women alone can complete this work in 7 days ?

A-6

B-9

C-5

D-7
Answer: D) 7

Explanation
Let 1 woman's 1 day work = x.

Then, 1 man's 1 day work = x/2 and 1 child's 1 day work x/4.

So, (3x/2 + 4x + + 6x/4) = 1/7

28x/4 = 1/7 => x = 1/49

1 woman alone can complete the work in 49 days.

So, to complete the work in 7 days, number of women required = 49/7 = 7.

Q 43 - A, B and C can complete a piece of work in 24,6 and 12 days


respectively.Working together, they will complete the same work in:

A - 1/24 days

B - 7/24 days

C - 24/7 days

D - 4 Days

Answer: C) 24/7 days

Explanation
(A+B+C)'s 1 day's work = (1/24 + 1/6 + 1/12) = 7/24

so, A,B and C together will complete the work in 24/7 days.
Q 44 - A and B together can do a piece of work in 40 days. A having worked for
20 days, B finishes the remaining work alone in 60 days. In How many days
shall B finish the whole work alone?

A - 60

B - 70

C - 80

D - 90
Answer: C) 80

Explanation

Let A's 1 day's work=x and B's 1 day's work=y

Then x+y = 1/40 and 20x+60y=1

Solving these two equations , we get : x= 1/80 and y= 1/80

Therefore B's 1 day work = 1/80

Hence,B alone shall finish the whole work in 80 days

Q 45 - A is 30% more efficient than B. How much time will they, working
together, take to complete a job which A alone could have done in 23 days ?

A - 9 Days

B - 11 Days

C - 13 Days
D - 15 Days

Answer: C) 13 days

Explanation

Ratio of times taken by A and B = 100 : 130 = 10 : 13.


Suppose B takes x days to do the work.

Then, 10 : 13 :: 23 : x => x = ( 23 x 13/10 ) => x = 299 /10.

A's 1 day's work = 1/23 ;


B's 1 day's work = 10/299 .

(A + B)'s 1 day's work = ( 1/23 + 10/299 ) = 23/299 = 113 .

Therefore, A and B together can complete the work in 13 days.

TRAINS

1. Speed in km/hr

a km /hr = (a * 5 / 18) m/s.

2. Speed in m/s

a m/s = ( a * 18/5) km/hr.


3. Time taken by a train of length L metres to pass a pole or a standing
man or a signal post is equal to the time taken by the train to cover

L Metres.

4. Time taken by a train of length L metres to pass a stationary object of


length b metres is the time taken by the train to cover

(L + b) metres.

5. Suppose two train or two bodies are moving in the same direction at u
m/s and v m/s , where u > v, then their

relative speed = (u – v) m/s.

6. Suppose two trains or two bodies are moving in opposite directions at u


m/s and v m/s , then their

relative speed = ( u + v) m/s.


7. If two trains of length a metres and b metres are moving in opposite
directions at u m/s and v m/s, then time taken by the trains to cross
each other =

(a+b) / ( u+ v) sec.

8. If two train s of length a metres and b metres are moving in the same
direction at u m/s and v m/s , then the time taken by the faster train to
cross the slower train =

( a+b) / (u + v ) sec.

9. If two train ( or bodies) start at the same time from points A and B
towards each other and after crossing they take a and b sec in reaching
B and A respectively, then

( A speed) : ( B speed) = ( √b : √a ).

Q1. An express Train of length 132m coming from the south direction having a speed
of 40 km/hr. What time it will take to cross an object on the coming platform.

A.11 sec

B.14 sec

C. 13 sec

D. 9 sec
Answer: A

Explanation:

Speed of the train = (40x )m/sec = 100/9 m/sec

Distance moved in passing the object = 132 m.

Required time taken = = 11.88 = 11 sec

Q2. An express Train of 0.145km long coming from the south direction crosses a
signal in time 15 sec. At what speed it crosses the signal.

A.9 m/sec

B.12 m/sec

C. 7 m/sec

D. 10 m/sec

Answer: A

Explanation:

Length of the train = 0.145 x 1000 m = 145 m

Distance moved in passing the object = 145 m.

Required speed taken = = 9.66 = 9 m/sec


Q3. AC emu local train is moving at a speed of 135 km/hr. Train has 15 cars each
24m long, what time will it take to cross the platform of length 120m?

A.12 m/sec

B.10 m/sec

C. 14 m/sec

D. 15 m/sec

Answer: A

Explanation:

Speed of the train = (135x )m/sec = 75/2 m/sec

Length of the train = 15 x 24 m = 360 m

Distance covered in passing the platform = (360 + 120) m = 480 m.

Required Time taken = = 12.8 = 12 m/sec

Q4. Find the speed of the local train in km/hr. If a train has 15 cars each 24m long
crosses the platform of length 120m in 15 second time?

A.32.0

B.62.4

C. 115.2

D. 120.5

Answer: A

Explanation
Length of the train = 15 x 24 m = 360 m

Distance covered in passing the platform = (360 + 120) m = 480 m.

Required Speed taken = = 32 m/s = 115.2 km/hr

Q5. Rahul was going to visit IPL 2022 at Wankhede Stadium, so he went to a railway
station to catch a train. The platform was 64m long and he observed that the train
passed the railway station in 18 second and also crossed him in 6 seconds. At what
speed the train was travelling.

A.6.24

B.9.66

C. 5.33

D. 7.62

Answer: C

Explanation

Let the length of the train be x metres

Then, the train covers x metres in 6 seconds and (x + 64) metres in 18 seconds.

= ⇒ x =32

So, the length of the train = 32 m.

Speed of the train = = 5.33 m/s

Q6. A train with 12 cars 12m each was coming from the north direction with a speed
of 74 kmph. What time will it cross the dog who is running at 12 kmph in the north
direction?
A. 13.04

B.6.43

C. 10.12

D. 8.36

Answer: D

Explanation

Length of the train = 12 x 12 m = 144 m

Speed of the train relative to dog = (74 – 12) kmph = 62 kmph

= (62x )m/sec = = 17.22 m/sec

Time taken by the train to cross the dog

= Time taken by it to cover 144 m at 17.22 m/sec = (144x )= 8.36 sec

Q7. A train with 12 cars 12m each was coming from the north direction with a speed
of 74 kmph. What time will it cross the dog who is running at 12 kmph in the south
direction?

A. 13.04

B.6.02

C. 10.12

D. 8.36

Answer: D

Explanation

Length of the train = 12 x 12 m = 144 m


Speed of the train relative to dog = (74 + 12) kmph = 86 kmph

= (86x )m/sec = = 23.88 m/sec

Time taken by the train to cross the dog

= Time taken by it to cover 144 m at 23.88 m/sec = (144x )= 6.02sec

Q8. If Two Trains X and Y having length 140 m and 210 m are running towards each
other on parallel lines, one at the rate of 70 kmph and another at 66 kmph. What
time will it take to cross each other?

A. 13.04 sec

B.9.26sec

C. 10.12 sec

D. 8.36 sec

Answer: D

Explanation

Relative speed of the two trains = 70+66 = 136 kmph

= (136x )m/sec = = 37.77 m/sec

Time taken by the trains to pass each other

= Time taken to cover (140 + 210) m at 37.77 m/sec = = 9.26 sec

Q9. Ritesh wanted to attain a meeting in thane, so he took a train and reached thane
station as he reached thane station a train crossed him, and he was walking along
the line in the same direction at the rate of 4 km/hr in 28 seconds. The train was
240m long. What was the speed of the train in km/hr?

A.42

B.26

C.34

D.

Answer: C

Explanation

Speed of the train relative to Ritesh = m/s = m/s

= x = km/hr.

Let the speed of the train be x km/hr. Then, relative speed = (x – 4) km/hr.

x–4= +4= = 34 km/hr

Q10. Tanuja wanted to attain a meeting in thane, so she took a train and reached
thane station as she reached thane station a train crossed her, and she was
walking in the opposite direction to the train at the rate of 4 km/hr in 28 seconds.
The train was 240m long. What was the speed of the train in km/hr?

A.42

B.26

C.34

D.
Answer: C

Explanation

Speed of the train relative to Ritesh = m/s = m/s

= x = km/hr.

Let the speed of the train be x km/hr. Then, relative speed = (x + 4) km/hr.

x+4= -4= = 26 km/hr

Q11. Durgesh was at a railway station walking at the rate of 4 km/hr. He saw the
train coming and he was walking opposite direction to the train. The train crossed
him at 13 sec and it was 120m long. Find the speed of the train?

A.8

B.34

C.28

D. 18

Answer: C

Explanation

Let the speed of the train be x km/hr.

Speed of the train relative to Durgesh = (x + 4) km/hr = (x + 4) x

= 15x + 60 = 480 x = 28

Hence, speed of the train = 28 km/hr


Q12. A Ladies special train of length 140m travels at the rate of 50 km/hr passing
another train and crosses another train in 12 seconds. What is the length of the
second train if it is travelling at twice the speed of the first train?

A.600

B.460

C.360

D. 500

Answer: C

Explanation

Relative speed = (50 + 100) km/hr = (150 x ) = 41.66 m/s

Let the length of the second train be x metres

Then, = 41.66

⇒ x + 140 =500

⇒ x = 360

Hence, length of second train = 360 m

Q13. A train enters a bridge 4 miles long at a rate of 45 mph. What time will it take
to pass through the bridge as soon as it enters the bridge and exits the bridge if the
train is ⅓ long?

A.8.32 min

B.10.24 min
C. 12.67 min

D. 6.22 min

Answer: D

Explanation

Total distance covered = ( + ) = 14/3 m/s

Time taken = ) = 0.1037 hrs

= (0.1037 x 60) = 6.22 min

Q14. Chennai express passed a railway station in 46 seconds and Shahrukh was
standing on the station in 24 seconds. If the speed of the train is 24 km/hr, what is
the length of the Railway station?

A.8.32

B.10.24

C.14.67

D. 146.67

Answer: D

Explanation

Speed = 24 km/hr = 24x (5/18) = 6.67 m/s

Length of the train = (24 x 6.67) = 160 m

Let the length of the platform be x metres.

⇒ = 6.67
⇒ x = 146.67

Q15. If two trains, one travelling with the speed of 80 km/hr cross a tunnel in 48 sec
and the second train which is 80m longer cross the same tunnel at 40 km/hr. Find
the time taken by the second train?

A. 20 sec

B.16 sec

C.25 sec

D. 30 sec

Answer: C

Explanation

Let the lengths of the train and the bridge be x metres and y metres respectively.

Speed of the first train = 80 km/hr = (80 x ) m/s = 22.22 m/s

Speed of the second train = 40 km/hr = (40 x ) m/s = 11.11 m/s

Then, = 11.11 ⇒ x+y = 256.86 ……. i

Required time = =

= = 15.9 sec= 16 Sec

Q16. A train enters a railway station 5 miles long at a rate of 30 mph. What time
will it take to pass through the railway station as soon as it enters the station and
exits, if the train is ¼ long?
A.15

B.23

C. 11

D. 17

Answer: C

Explanation

Total distance covered = ( + ) = 22/4 m/s

Time taken = ) = 0.1833 hrs

= (0.1833 x 60) = 11 min

Q17. A girl was sitting on the window seat of the train and started counting trees.
The distances between the two trees were 10 metres, in 2 hours how many trees
would she count if the train was moving with the speed of 40 km/hr?

A.8000

B.7500

C.8001

D.7501

Answer: C

Explanation

Distance covered by the train in 2 hours

= (40 × 2) km = 80 km = 80000 m
⇒ Number of trees counted by the girl = ( + 1) = 8001.

Q18. Two trains having length 70m and 90m respectively. If the first train overtakes
another in the same direction in 0.276 minutes. At what speed the first train is
moving when the second train is travelling at 40 kmph?

A. 74.78

B.64.62

C. 82.32

D. 92.41

Answer: A

Explanation:

Let the speed of the first train be x km/hr.

Then, sum of lengths of trains = (70 + 90) m = 160 m

Relative speed of two trains = (x – 40) kmph = (x - 40) x m/s

= 0.276 x 60 5(x-40) =

x= 74.78 km/hr

Q19. Prachet was sitting in a train which was moving at 40 kmph and observed that
an Ac emu train, travelling in the opposite direction, took 12 seconds to cross him if
the train is 240m long, find its speed?

A.48

B.27

C. 50
D. 32

Answer: D

Explanation:

Relative speed = ( )m/s = ( ) kmph = 72 kmph

∴ Speed of Ac train = (72 – 40) kmph = 32 kmph.

Q20. Hyderabad express and Mumbai express train start moving at the same time
from their respective station and proceed toward each other at the rate of 25 m/s
and 20 m/s respectively. When both meet at a point in between, one train was found
to have travelled 240 km more than the other. Find the distance between the
stations.

A.2160

B.2210

C. 2340

D. 2024

Answer: A

Explanation

Let the trains meet after t hours.

Speed of train Hyderabad express = 25 m/s

Speed of train Mumbai express = 20 m/s

Distance covered by train Hyderabad = 25 × t = 25t

Distance covered by train Mumbai = 20 × t = 20t


Distance = Speed × Time

According to the question,

25t – 20t = 240

⇒ 5t = 240

⇒ t = 48

Distance between station = 25t + 20t = 45t

= 45 × 48 = 2160 km

Q21. Samarth was going to visit Rani baugh at Byculla, so he went to a railway
station to catch a train. The platform was 80m long and he observed that the train
passed the railway station in 16 second and crossed him in 8 seconds. At what
speed the train was travelling.

A.12

B.9

C. 10

D. 7

Answer: C

Explanation

Let the length of the train be x metres

Then, the train covers x metres in 8 seconds and (x + 80) metres in 16 seconds.

= ⇒ x =80

So, the length of the train = 80 m.


Speed of the train = = 10 m/s

Q22. Soham wanted to attain a meeting at Rajasthan, so he took a train and reached
Rajasthan station. As he reached the station a train crossed him, and he was
walking along the line in the same direction at the rate of 6 km/hr in 24 seconds.
The train was 280m long. What was the speed of the train in m/s?

A.50

B. 27

C.48

D.

Answer: D

Explanation

Speed of the train relative to Soham = m/s = m/s

= x = 42 km/hr.

Let the speed of the train be x km/hr. Then, relative speed = (x – 6) km/hr.

x–6= + 6 = 48 km/hr = 48 x = 13.33 m/s

Q 23 - What is 90 kmph as metres per second?

A - 15 m /sec

B - 20 m /sec

C - 25 m /sec
D - 30 m /sec

Answer - C

Explanation

90 kmph = ( 90 * 5/18) m/sec = 25 m /sec.

Q 24 - What is 35 m/sec as km/hr?

A - 123 km/hr

B - 124 km/hr

C - 125 km/hr

D - 126 km/hr

Answer - D

Explanation

35 m/sec = (35 * 18 / 5 ) km/hr = 126 km/hr.

Q 25 - A 75m long train is running at 54 km/hr. In how much time will it cross
an electric pole?

A - 25 sec
B - 20 sec

C - 15 sec

D - 5 sec

Answer - D

Explanation

Speed of the train = ( 54 * 5 / 18) m/sec = 15 m / sec.

Time taken to cross an electric pole = Time taken to cover 75m

= ( 75 / 15 ) sec = 5 sec.

Q 26 - A 415 m long train is running at 63 km/hr. In how much time will it


cross a tunnel 285 m long?

A - 40 sec

B - 50 sec

C - 60 sec

D - 70 sec

Answer - A
Explanation

Speed of the train= ( 63 * 5 / 18) m/sec = 35/2 m/sec.

Time taken to cross the tunnel = Time taken to cover ( 415 + 285) m

= ( 700 * 2/35 ) sec = 40 sec.

Q 27 - A train passes a standing man in 3 seconds and a platform 105 m long


in 8 seconds. Find the length of the train and its speed?

A - 59 m, 75.6 km/hr

B - 61 m, 72.6 km/hr

C - 63 m, 75.6 km/hr

D - 66 m, 79.6 km/hr

Answer - C

Explanation

Let the length of the train be x metres and its speed be y km/hr i.e. (5y/18 )
m/sec.

Then, x / ( 5y / 18 ) = 3 ⇒ 18x = 15y ⇒ 6x = 5y.

Also, ( x + 105) / ( 5y/ 18) = 8 ⇒ 18 ( x + 105 ) = 40y⇒ 9 ( x + 105 ) = 20y


⇒ 20y -9x = 945 ⇒ 24x – 9x = 945 ⇒ 15x = 945 ⇒ x= 63.

∴ 5y = ( 6 * 63 ) ⇒ y = ( 6 * 63 ) /5 = 378 / 5 = 75.6

Hence, the length of the train is 63 m and its speed is 75.6 km/hr.

Q 28 - A train 125m long is running at 50 km/ hr. In what time will it pass a
man , running at 5 km/hr in the same direction in which the train is going?

A - 22 sec

B - 20 sec

C - 15 sec

D - 10 sec

Answer - D

Explanation

Speed of the train relative to man = ( 50 – 5 ) km/hr

= ( 45 * 5 / 18) m/sec = 25/2 m/ sec.

Distance covered in passing the man = 125m.

∴ Time taken = 125 / ( 25/2) sec = ( 125 * 2 / 25) sec = 10 sec.


Q 29 - A train 110 m long is running at 60 km / hr. In what time will it pass a
man, running in the direction opposite to that of the train at 6 km/hr?

A - 9 sec

B - 8 sec

C - 7 sec

D - 6 sec

Answer - D

Explanation

Speed of the train relative to man = ( 60 + 6 km/hr = 66 km/hr

= ( 66 * 15 / 18 ) m/sec = 55/3 m/sec.

Distance covered in passing the man = 110m.

Time taken = 110//( 55/3) sec = ( 110 * 3 / 55 ) sec = 6 sec.

Q 30 - A train 100m long takes 9 seconds to cross a man walking at 5 km/hr in


the direction opposite to that of the train. Find the speed of the train.

A - 55 km/hr

B - 45 km/hr
C - 25 km/hr

D - 35 km/hr

Answer - D

Explanation

Let the speed of the train be x km/hr.

Relative speed = ( x + 5 ) km /hr = 5 ( x+ 5 ) / 18 m/sec.

Distance covered in passing the man = 100m.

∴ 100/ 5 ( x+5) / 18 = 9 ⇒ 45 ( x+ 5) = 1800 ⇒ x + 5 = 40 ⇒ x = 35.

Speed of the train = 35 km/hr.

Q 31 - Two train 128 m and 132m long are running towards each other on
parallel lines at 42 km/hr and 30 km / hr respectively . In what time will they
be clear of each other from the moment they meet?

A - 13 sec

B - 14 sec

C - 15 sec

D - 16 sec
Answer - A

Explanation

Relative speed = ( 42 + 30 ) km/hr = 72 km/hr

= ( 72 * 5 / 18 ) m/sec = 20 m / sec.

Distance covered in passing each other = ( 128 + 132) m = 260m.

∴ Required time = 260 / 20 sec= 13 sec.

Q 32 - A man's speed with the current is 15 km/hr and the speed of the
current is 2.5 km/hr. The man's speed against the current is :

A - 9.5 Km / Hr

B - 10 Km / Hr

C - 10.5 Km / Hr

D - 11 Km / Hr

Answer: B) 10 km/hr
Explanation

Man's rate in still water = (15 - 2.5) km/hr = 12.5 km/hr.

Therefore, Man's rate against the current = (12.5 - 2.5) = 10 km/hr.


Q 33 - A man sitting in a train which is traveling at 50 kmph observes that a
goods train, traveling in opposite direction, takes 9 seconds to pass him. If the
goods train is 280 m long, find its speed.?

A - 60

B - 62

C - 64

D - 65

Answer: B) 62
Explanation

Relative speed =280/9 m / sec = (280/9*18/5) kmph = 112 kmph.

Speed of goods train = (112 - 50) kmph = 62 kmph.

Q 34 - Two cogged wheels of which one has 32 cogs and other 54 cogs, work
into each other. If the latter turns 80 times in three quarters of a minute, how
often does the other turn in 8 seconds?

A - 48

B - 24

C - 38
D - 36

Answer: B) 24
Explanation

Less Cogs more turns and less time less turns

Cogs Time Turns

A 54 45 80

B 32 8 ?

Number of turns required=80 × 54/32 × 8/45 = 24 times

Q 35 - Two stations P and Q are 110 km apart on a straight track. One train
starts from P at 7 a.m. and travels towards Q at 20 kmph. Another train starts
from Q at 8 a.m. and travels towards P at a speed of 25 kmph. At what time
will they meet?

A - 10.30

B - 10

C - 8.45

D - 9.30
Answer: B) 10
Explanation
Assume both trains meet after x hours after 7 am
Distance covered by train starting from P in x hours = 20x km
Distance covered by train starting from Q in (x-1) hours = 25(x-1)
Total distance = 110

=> 20x + 25(x-1) = 110


=> 45x = 135
=> x= 3
Means, they meet after 3 hours after 7 am, ie, they meet at 10 am

Q 36 - A train speeds past a pole in 15 seconds and a platform 100 m long in 25


seconds. Its length is:

A -50 m

B - 150 m

C - 200 m

D - Data Inadequate

Answer: B) 150 m
Explanation
Let the length of the train be x metres and its speed be y m/sec.
Then, (x/y)= 15 y =(x/15)

(x+100)/25 = x/15

=> 15(x + 100) = 25x

=> 15x + 1500 = 25x

=> 1500 = 10x

=> x = 150 m.

Q 37 - A 270 metres long train running at the speed of 120 kmph crosses
another train running in opposite direction at the speed of 80 kmph in 9
seconds. What is the length of the other train ?

A - 230 m

B - 240 m

C - 260 m

D - 320 m

Answer: A) 230 m
Explanation

Relative speed = (120 + 80) km/hr


=(200*5/18)m/s = (500/9)m/s

Let the length of the other train be x metres.

Then, x+270/9 = 500/9

=> x + 270 = 500

=> x = 230m

Q 38 - A train 800 metres long is running at a speed of 78 km/hr. If it crosses a


tunnel in 1 minute, then the length of the tunnel (in meters) is :

A - 130

B - 360

C - 500

D - 540

Answer: C) 500
Explanation
Speed =[ 78 x ( 5/18 ) ] m/sec = 65/3 m/sec.

Time = 1 minute = 60 sec.

Let the length of the tunnel be x metres.

Then, [ (800 + x )/ 60 ]= 65/3

3(800 + x) = 3900

x = 500.

Q 39 - Two trains are moving in the same direction at 72 kmph and 36 kmph.
The faster train crosses a girl sitting at window seat in the slower train in 32
seconds. Find the length of the faster train ?

A - 170 m

B - 100 m

C - 270 m

D - 320 m

Answer: D) 320 m

Explanation
Relative speed = (72 - 36) x 5/18 = 2 x 5 = 10 mps.
Distance covered in 32 sec = 32 x 10 = 320 m.

The length of the faster train = 320 m.

Q 40 - Two trains of equal length, running with the speeds of 60 and 40 kmph,
take 50 seconds to cross each other while they are running in the same
direction. What time will they take to cross each other if they are running in
opposite directions ?

A - 10 sec

B - 11 sec

C - 12 sec

D - 8 sec

Answer: A) 10 sec

Explanation
Relative Speed = 60 -40 = 20 x 5/18 = 100/18

Time = 50

Distance = 50 x 100/18 = 2500/9


Relative Speed = 60 + 40 = 100 x 5/18

Time = 2500/9 x 18/500 = 10 sec.

Q 41 - Two goods trains each 520 m long, are running in opposite directions on
parallel tracks. Their speeds are 42 km/hr and 36 km/hr respectively. Find the
time taken by the slower train to cross the driver of the faster one ?

A - 60 sec

B - 48 sec

C - 45 sec

D - 34 sec

Answer: B) 48 sec
Explanation
Relative speed = 42 + 36 = 78 km/hr = 65/3 m/s

Distance = (520 + 520) =1040 mts.

Time = 1040 x 3/65= 48 sec

Q 42 - A train for Fathehpur leaves for every 2 hrs 30 min from Agra station. An
announcement was made that train left 37 mins ago and next train comes at
17:00hrs. At what time was the announcement made ?
A - 15:07 Hrs

B - 15:20 Hrs

C - 15:05 Hrs

D - 15:00 Hrs

Answer: A) 15:07 hrs


Explanation

Next train comes at 17:00 hrs.


So, last train will be = 17:00hrs - 2:30hrs
= 14:30hrs
Announcement made after 37 min of the last train.
S0, 14:30hrs + 00:37 min
= 15:07 hrs.
HCF & LCM
Factors and Multiples

If a number P divides another number Q exactly, we say that P is a factor of Q


i.e. Q is a multiple of P.

H.C.F

The H.C.F of two or more than two numbers is the greatest number that
divides each of them exactly.

L.C.M

The least number which is exactly divisible by each one of the given numbers
is called their L.C.M

Product of two numbers

Product of their H.C.F and L.C.M

Co-primes

Two numbers are co-primes if their H.C.F is 1.

H.C.F and L.C.M of fractions

1. H.C.F = H.C.F of Numerators⁄L.C.M of Denominators


2. L.C.M = L.C.M of Numerators⁄H.C.F of Denominators
Q 1 - Compute H.C.F of (22* 23*5*74), (23*32*52*73) and (22*53*75).
A - 6760
B - 6860
C - 6960
D - 7060

Answer - B
Explanation
Prime numbers which are common to all the given numbers are 2,5 ,7.
∴ H.C.F = (22*5*73)= (4*5*343) = 6860

Q 2 - Find the H.C.F of 108, 360 and 600.


A - 12
B - 13
C - 14
D - 15

Answer - A
Explanation
108 = (22*33) , 360 = (23*32*5) and 600 = (23*52*3)
∴ H.C.F = (22* 3) = (4* 3)=12

Q 3 - Find the H.C.F of 148 and 185.


A - 37
B - 38
C - 39
D - 40

Answer - A
Explanation
Remainder of 185/148 = 37
Remainder of 148/37 = 0
∴ H.C.F. = 37

Q 4 - Find the H.C.F of 204, 1190 and 1445.


A - 16
B - 17
C - 18
D - 19
Answer - B
Explanation
Remainder of 1190/204 = 170
Remainder of 204/170 = 34
Remainder of 170/34 = 0

∴ H.C.F. of 204, 1190 = 34


Remainder of 1145/34 = 17
Remainder of 34/17 = 0

∴ H.C.F. of 204, 1190 and 1145 = 17


Q 5 - Reduce 391/667 to lowest terms.
A - 7/29
B - 27/29
C - 17/29
D - 37/29

Answer - C
Explanation
First we find the H.C.F of 391 and 667.
Remainder of 667/391 = 276
Remainder of 391/276 =115
Remainder of 276/115 = 46
Remainder of 115/46 = 23
Remainder of 46/23= 0

∴ H.C.F. of 391, 667 = 23


∴ 391/667 =( 391/23)/ (667/23)= 17/29

Q 6 - Find the L.C.M of (22*32*5*7) , (2³*3*52*72) and (2*3*7*11).


A - 970200
B - 97020
C - 9702
D - 970

Answer - A
Explanation
We have L.C.M = product of terms containing highest powers of (2,3,5,7,11)
= (2³* 32* 52*72*11) = (8*9*25*11*49)= 970200

Q 7 - Find the L.C.M of 15, 18, 24, 27, 56.


A - 7260
B - 7360
C - 7460
D - 7560
Answer - D
Explanation
15 = 3 * 5
18 = 2* 3 * 3 = 2 * 3 2
24 = 2 *2 * 2 * 3 = 2 3 * 3
27 = 3 * 3 * 3 = 3 3
56 = 2 * 2 * 2 * 7= 2 3 * 7
L.C.M = product of terms containing highest powers of (2,3,5,7) =2 3 * 3 3 * 5 *
7 = 7560

Q 8 - Find the H.C.F and L.C.M of 2/3 , 8/9 , 10/27 and 16/81.
A - 2 / 84 , 80/ 9
B - 5/4 , 75/3
C - 5/2 , 81/4
D - 2/81 , 80/3

Answer - D
Explanation
H.C.F of 2,8,10,16 = 2
L.C.M of 3,9,27,81 = 81
H.C.f = H.C.F of 2,8,10,16/L.C.M of 3,9,27,81 = 2/81
L.C.M = L.C.M of 2,8,10,16/H.C.F of 3,9, 27,81 = 80/3

Q 9 - Two numbers are in the ratio 8:11 . Considering their H.C.f as 6, find the
numbers.
A - 58 , 79
B - 48 , 66
C - 38 , 56
D - 28 , 33

Answer - B
Explanation
Let the numbers be 8x and 11 x. then, their H.C.F = x
So, the numbers are (8*6), (11*6) i.e 48 and 66.

Q 10 - Given the H.C. F of two numbers as 7 and their L.C.M as 210. If one of
the numbers is 35, find the other.
A - 32
B - 42
C - 52
D - 62
Answer - B
Explanation
Let the Other number be X. then,
Product of numbers = product of their H.C .F and L.C.M
35*x= 7* 210 ⇒ x= 7*210/35 = 42
Hence, the other number is 42.

Q 11 - Three big drums contain 36 liters, 45 liters and 72 liters of oil. What is
the biggest measure which can measure all the different quantities exactly?
A - 9 liters
B - 10 liters
C - 11 liters
D - 12 liters

Answer - A
Explanation
Required measure = H.C.F of 36 L, 45 L, and 72 L
= (32) liters = 9 liters
[As 36 = 22*32, 45 = 32*5 and 72 = 24* 34]

Q 12 - Four electronic devices make a beep after duration of 30 minutes, 1


hour, 3/2 hours and 1 hour 45 min. respectively. If all the devices beeped
together at 12 noon at what time will they beep together again?
A - 9 am
B - 10 am
C - 11 am
D - 11:30 am

Answer - A
Explanation
Intervals of beeping 30 min, 60 min, 90 min, 105 min.
Interval of beeping together= L.C.M of 30 min. 60 min. 90 min. 105 min
= (3*5*2*2*3*7) min. = 1260 min = 21 hrs.
So, they will beep together again next morning at 9 am.

Q 13 - Find the largest number which can exactly divide 513, 783 and 1107.
A - 22
B - 23
C - 24
D - 25

Answer - B
Explanation
Remainder of 783/513 = 270
Remainder of 513/270 =243
Remainder of 270/243 = 27
Remainder of 243/27 = 0
Remainder of 46/23= 0
∴ H.C.F. of 513, 783 = 23
Remainder of 1107/23 = 0
∴ H.C.F. of 513, 783 and 1107= 23
Q 14 - Find the smallest number which is exactly divisible by each one of the
numbers 12, 15, 20 and 27.
A - 540
B -530
C - 520
D - 510

Answer - A
Explanation
Required no. = L.C.M of 12,15, 20 and 27
= (3*2*2*5*9) = 540

Q 15 - Find the least number which if divided by 6, 7, 8, 9, 12 leaves the same


remainder 2 in each case.
A - 506
B - 504
C - 502
D - 500
Answer - A
Explanation
Required number = (L.C.M of 6,7,8,9,12)+2 = (2*3*2*7*2*3)+ 2 = (504+2)=
506.

Q 16 - Find the largest natural number which can divide the product of any 4
consecutive natural numbers.
A - 23
B - 24
C - 25
D - 26

Answer - B
Explanation
(1*2*3*4) = 24
∴ Required number = 24

Q 17 - Find the least number which if divided by 35, 45 and 55 leaves the
remainder 18, 28 and 38 respectively.
A - 3448
B - 3458
C - 3468
D - 3478
Answer - A
Explanation
Here (35-18) = 17 , (45-28)= 17 and (55- 38) = 17
Required number = (L.C.M of 35,45, 55)- 17 = (3465 -17) = 3448

Q 18 - The H.C.F of 1/2 , 2/3 , 3/4 , 4/5 is


A - 1/120
B - 12/5
C - 100/3
D - 10/3

Answer - A
Explanation
H.C.F = H.C.F of 1,2,3,4/ L.C.M of 2,3,4,5 = 1/120

Q 19 - The H.C.F of 2/3, 8/9 , 10/27 , 32/81.


A - 160/81
B - 160/3
C - 2/81
D - 2/3

Answer - C
Explanation
H.C.F = H.C.F of 2, 8,10, 32/ L.C.M of 3,9, 27, 81 = 2/81

Q 20 - Which of the following is a pair of Co-primes?


A - (14, 35)
B - (18, 25)
C - (31, 93)
D - (32,62)
Answer - B
Explanation
H.C.F of 18 and 25 is 1.
∴ 18 and 25 are co-primes.

Q 21 - Find the L.C.M of 72, 108, and 2100.


A - 37500
B - 37000
C - 37800
D - 37850

Answer : C

Explanation

72 = 23 x 32
108 = 22 x 33
2100 = 22 x 52 x 3 x7
Therefore L.C.M = 2 3 x 33 x 52 = 37800

Q 22 - The smallest fraction, which each of 6⁄7, 5⁄14, 10⁄21 will divide exactly, is?
A - 38⁄98
B - 60⁄147
C - 50⁄294
D - 30⁄7

Answer : D

Explanation
Required fraction = L.C.M of 6⁄7, 5⁄14, 10⁄21 = L.C.M of 6, 5, 10⁄H.C.F of 7, 14, 21 = 30⁄7

Q 23 - Find the highest common factor of 36 and 84.


A-5
B-6
C-4
D - 12

Answer : D

Explanation

36 = 22 x 32 84 = 22 x 3 x 7.
Therefore H.C.F = 22 x 3 = 12.

Q 24 - If the sum of two numbers is 55 and the H.C.F and L.C.M of these
numbers are 5 and 120 respectively then the sum of the reciprocals of the
numbers is equal to?
A - 11⁄120
B - 55⁄601
C - 601⁄55
D - 120⁄11

Answer : A

Explanation

Let the numbers be a and b. Then a + b = 55 and a x b = 5 x 120 = 600


therefore Required sum = 1⁄a + 1⁄b = a + b⁄a x b
55
⁄600 = 11⁄120
Q 25 - Let N be the greatest number that will divide 1305, 4665 and 6905
leaving the same remainder in each case. then the sum of the digits in N is?
A-5
B-4
C-6
D-8

Answer : B

Explanation

N = H.C.F of (4665 - 1305), (6905 - 4665), (6905 - 1305)


= H.C.F of 3360, 2240 and 5600 = 1120.
Sum of digits in N = (1 + 1 + 2 + 0) = 4.
Q 26 - If the L.C.M of A and B is C, their H.C.F. is
A - C/AB
B - ABC
C - (A+B)/C
D - AB/C

Answer : D

Explanation

HCF*LCM=A*B
⇒HCF=(A*B)/LCM=AB/C

Q 27 - Find the greatest possible rate at which a man should walk to cover a
distance of 35 km and 140 km in exact number of days
A - 30
B - 70
C - 25
D - 35

Answer : D

Explanation

GCD of two 35 and 140=35

Q 28 - If two numbers are greater than 13 and the H.C.F of two numbers be
13, L.C.M 273, then the sum of the numbers is:
A - 286
B - 130
C - 288
D - 290

Answer : B

Explanation

Let the number be 13 a and 13 b, where a and b are co-primes.


Then, 13a * 13b= (13* 273) ⇒ab= 21
Two co-primes with product 21are 3 and 7.
∴ numbers are (13*3, 13*7) i.e , 39 and 91.
Their sum = (39+91) = 130

Q 29 - If the H.C.F of two numbers and their difference is 12 find the numbers.
A - 66, 78
B - 70, 82
C - 94, 106
D - 84, 96
Answer : D

Explanation

The difference of requisite numbers must be 12 and one must be divisible by


12 .
Hence, the required numbers are 84 and 96

Q 30 - If the product of two co-primes is 117 then their L.C.F should be


A-1
B - 117
C - equal to their H.C.F
D - cannot be calculated

Answer : B

Explanation

H.C.F of co-primes=1
H.C.F * L.C.M = Their product = 117
∴ 1* L.C.M = 117 ⇒L.C.M = 117
SIMPLE INTEREST

1. Simple Interest : If the interest on a sum of money borrowed for a


certain time period is reckoned uniformly, it is called as Simple Interest.

2. If Principal = P, Rate = R% per annum, Time = T years, then

Simple Interest (S.I) = P * T * R / 100

The above expression involves four quantities: P, I, T and R. If three of


them are known to us,the fourth one can be easily found out.

3. Amount (A) = Principal(P) + Interest(I)


We can correlate it as

A = P + P * T * R/100

Or,

A = P(1 + T * R/100)

4. When time is given in days, convert it into years by dividing by 365.


Similarly, if given in months, convert it into years by dividing it by 12.

5. When type of interest is not specified, it is assumed to be simple


interest.

6. Rate of interest is taken per annum unless specified otherwise.


7. The day when money is deposited or borrowed is omitted while the day
on which money is withdrawn or paid back is counted in calculating
time period.

Q1. Sonali has borrowedRs. 5,000 from her relative on 2nd March 2022 at 6 % per
annum and will return on 22ndMay 2022. Find the simple interest.
A.75.45

B.82.41

C. 67.30

D. 59.86

Answer: A

Explanation:

Time = (29 + 30 +22) =

P = Rs 5000, R = %, T = year

∴ S.I. = = 75.45

Q2. A sum of Rs 700 amounts to Rs 820 in 4 years at simple interest. If the interest
rate is increased by 4%, it would amount to how much?

A.8.29%

B.9.24%

C.7.83%
D.9.42%

Answer: A

Explanation:

Simple Interest for 4 years = 820-700=120

S. I= P*T*R

120=700*4*R

R=120/2800=4.29%

Revised interest rate= 4+4.29=8.29%

Q3. If the simple interest accrued on amount of Rs 3000 at the end of 8 years is Rs
2450. Then, what would be the simple interest accrued on an amount of Rs. 8640 at
the same rate and for the same period.

A.6143

B.7056

C. 5842

D. 8465

Answer: B

Explanation:

P = Rs 3000, T = 8 years, SI = 2450

⇒ Rate = % = 10 %

Now, P = Rs 8640, T = 8 years, R = 10 %


SI = = Rs 7056

Q4. Radha took a loan from a bank which is near to her house at the rate of 4% for
the first three years, at the rate of 5% p.a. for the next four years, and at the rate of
12% p.a. For the period beyond seven years. If the total interest she paid is 12600 at
the end of twelve years, how much money she had lent?

A.21415.98

B.13695.65

C. 24013.34

D. 15521.17

Answer: B

Explanation:

Let the sum borrowed be x. Then,

+ + = 12600

+ + = 12600

= 12600

x= 13695.65

Q5. Raju invested a certain sum of money in a fixed deposit which becomes three
times of itself in 24 years at a simple interest. In how many years does it become
twice of itself at the same rate of simple interest?

A.10
B.16

C. 22

D. 18

Answer: B

Explanation:

Let principal = P. Then, S.I. = 3P and T = 24 yrs.

Rate = % = 12.5%

Now, principal = P, S.I. = 2P, R = 12.5%.

Time = yrs =16 years

Q5. The simple interest on a sum of money is 8/3 of the principal. Find the rate
percent and time if the rate of interest is double the time.

A.23.08 and 11.54

B.18.12 and 9.06

C. 25.00 and 12.5

D. 28.16 and 14.08

Answer: A

Explanation:

Let the sum = Rs x. Then, S.I =

Let rate = 2T% and time = T years.

Then, = ⇒ T² =
⇒ T = 11.54 years and Rate = 23.08%

Q6. Rate of interest had been put on 3% higher rate,it would have got Rs 540 more.
find the sum. if the sum was put at a simple interest rate at a certain rate for 5
years.

A. 1800

B.4200

C. 2400

D. 3600

Answer: D

Explanation:

Let sum = P and original rate = R.

Then, - = 540

⇔5PR + 15P – 5PR = 54000

⇔15P = 54000

⇔P = 3600.

Hence, sum = Rs 3600.

Q7. A Student borrows out Rs 15000 from slice app on the condition that the loan is
payable in 12 months by 12 monthly installments of 1500 each. Find the rate of
simple interest charged.

A.20%

B.8.1%
C.7.0%

D. 8.2%

Answer: B

Explanation:

We have:

Simple Interest =Amount repaid- Rs 15000

= (1500*12)-15000

=3000

Simple Interest = 15000*1 year* R%

R =3000/15000

R =20%

Q8. Raju borrowed 10000 Rs from his friends on 12nd February 2022 at 8 % per
annum and will return on 25 June 2022. Find the simple interest.

A.318.84

B.421.24

C. 246.31

D. 572.72

Answer: A

Explanation:

Time = (16 + 31 +30 +31 + 25) =


P = Rs 10000, R = %, T = year

∴ S.I. = = 318.84

Q9. Mahi borrowed x amount of money from his colleague on 18 Jan 2019 at the rate
of 9 % per annum and will return on 3 May 2019 and the simple interest was 327.94.
Find x?

A.13000

B.12000

C. 16000

D. 15000

Answer: B

Explanation:

Time = (13 + 28 + 31 +30 + 3) =

P = x, R = %, T = year

∴P= = 12000

Q10. If Sanjay put the sum of money at a simple interest rate at a certain rate for 5
years. Rate of interest had been put on 5% higher rate, it would have got Rs 640
more. Find the sum?

A. 1280

B.4200
C. 2560

D. 3600

Answer: C

Explanation:

Let sum = P and original rate = R.

Then, - = 640

⇔5PR + 25P – 5PR = 64000

⇔5P = 64000

⇔P = 2560.

Hence, sum = Rs 2560.

Q11. Rakesh took a personal loan from a bank which is near to his house at the rate
of 6% for the first two years, at the rate of 4% p.a. for the next five years, and at the
rate of 10% p.a. For the period beyond seven years. If the total interest he paid is
15000 at the end of Twelve years, how much money he borrowed?

A.21415.6

B.18292.7

C. 24707.2

D. 17504.4

Answer: B

Explanation:

Let the sum borrowed be x. Then,


+ + = 15000

+ + = 15000

= 15000

x= 18292.7

Q12. The numerical value of rate % and time is same and S.I on the sum of money is
7/2 of the principal. Then find the rate of interest.

A.2 %

B.5 %

C.6 %

D.3 %

Answer: B

Explanation:

Let the sum = Rs x. Then, S.I =

Let rate = R% and time = R years.

Then, = ⇒ R² =

⇒ R = 18.708 = 5 %

⇒R = 5 %

Q13. Satish borrowed Rs 15000 for 3 years at 2 % p.a simple interest. He


immediately Lends this money to his friend at 8 ⅔ % p.for 3 years. Find his profit in
the transaction per year.
A. 2000

B.3000

C. 1000

D. 4000

Answer: C

Explanation:

Profit in 2 years = -

= 3900 - 900

= 3000

Profit in 1 year = 3000/3 = 1000

Q14. Keshav borrowed 2400 partly at a rate of4% & 6% p.a. simple interest. The total
interest received after 4 years at the was Rs 400. The ratio of the money lent at 4%
to that lent 6% is.

A.33: 3

B.5: 55

C. 11: 1

D.7: 77

Answer: C

Explanation:

Let the sum lent at 4% be Rs x and that lent at 6% be (2400-x). then,

Interest on x at 4% for 4 years + interest on (2400-x) at 6% for 4 years = 400

+ = 400
X =2200

Required Ratio = x: (2400-x) = 2200: 200

⇒ 11: 1

Q15. A man invested Rs 10000 in a business at some rate of simple interest and
6000 at 2% higher rate of interest. If the interest in both the cases after 6 years is
the same, the rate of interest in the former case is.

A. 6%

B.4%

C. 5%

D. 3%

Answer: D

Explanation:

Let the rates of interest in the former and latter cases be R% and (R + 1) % p.a.

Then, 10000 × R × 6 = 6000 × (R + 2) × 6

60000R = 36000R + 72000

24000R = 72000

R = 3%

Q16. Rahul invested Rs x in dairy farming at a rate of 3% simple interest and Rs 6000
at 2% higher rate of interest. If the interest in both the cases after 6 years is the
same, Find the value of x.

A. 12000

B.9000
C. 8000

D. 10000

Answer: D

Explanation:

Let the rates of interest in the former and latter cases be 3% and (3 + 2) % p.a.

Then, x × 3% × 6 = 6000 × (3 + 2) % × 6

18x = 180000

x = 180000/18

x = 10,000

Q17. The numerical value of rate % and time is the same and simple interest on the
sum of money is 8/3 of the principal. Then find the rate of interest.

A.14.33 %

B.15.67%

C.16.32%

D.14.87%

Answer: C

Explanation:

Let the sum = Rs x. Then, S.I =

Let rate = R% and time = R years.

Then, = ⇒ R² =

⇒ R = 16.32
⇒ R = 10 %

Q18. Ganesh took a personal loan from a bank which is near to his house at the rate
of 5% for the first one years, at the rate of 3% p.a. for the next five years, and at the
rate of 8% p.a. for the period beyond six years. If the total interest he paid is 7000 at
the end of ten years, how much money he borrowed?

A.16123.12

B.13461.53

C.17180.17

D.18042.01

Answer: B

Explanation:

Let the sum borrowed be x. Then,

+ + = 7000

+ + = 7000

= 7000

x= 13461.53

Q19. Shiva lent 3400 partly at a rate of 3% &7% p.a. simple interest. The total interest
received after 5 year was Rs 800. The ratio of the money lent at 3% to that lent 7%
is.

A.69: 59

B.49: 39

C. 39: 29
D.29:19

Answer: C

Explanation:

Let the sum lent at 3% be Rs x and that lent at 7% be (800-x). then,

Interest on x at 3% for 5 years + interest on (3400-x) at 7% for 5 years = 800

+ = 800

X =1950

Required Ratio = x: (3400-x) = 1950: 1450

⇒ 39: 29

Q20. Mangesh borrowed Rs 18000 for 2 years at 1% p.a simple interest. He


immediately Lends this money to Manish at 6 ½ % p.for 2 years. Find his profit in the
transaction per year.

A.940

B.970

C.990

D.1980

Answer: C

Explanation:

Profit in 2 years = -

= 2340 - 360

= 1980
Profit in 1 year = 1980/2 = 990

Q21. If the simple interest increases over a period of time, an amount of Rs 4500 at
the end of 6 years is 2800. Then what would be the simple interest accrued on an
amount of Rs. 11340 at the same rate and for the same period.

A.6143

B.7056

C. 5842

D. 8465

Answer: B

Explanation:

P = Rs 4500, T = 6 years, SI = 2800

⇒ Rate = % = 10.37%

Now, P = Rs 11340, T = 6 years, R = 10.37%

SI = = Rs 7056

Q22. If Mangesh puts the sum of money at a simple interest rate at a certain rate for
4 years. Rate of interest had been put on 4% higher rate, it would have got Rs 720
more. If the product of sum and find the sum?

A. 2440

B.8410

C. 4500
D. 1200

Answer: C

Explanation:

Let sum = P and original rate = R.

Then, - = 720

⇔4PR + 16P – 4PR = 72000

⇔16P = 72000

⇔P = 4500.

Hence, sum = Rs 4500.

Q23. Mahesh borrowed Rs 7040 from his friend on 26th august 2018 at 7 % per
annum and will return on 23november 2018. Find the simple interest.

A.133.03

B.146.28

C.111.87

D.119.86

Answer: A

Explanation:

Time = (5+30+31 +23) =

P = Rs 7040, R = %, T = year
∴ S.I. = = 133.03

Q24. Some students invested money in a government scheme, a certain sum of


money amounts to Rs 1420 in 3 years and to Rs 1615 in 4.5 years. Find the money
invested and rate of interest.

A.1124 and 16.14 %

B.1030 and 12.62 %

C.980 and 12.34%

D.1120 and 15.11%

Answer: B

Explanation:

S.I for 1.5 years = Rs (1615 - 1420) = 195.

S.I for 3 years = 390

Principle = (1420 - 390) = 1030

Now, P = 1030, T = 3, S.I = 390

Rate = = 12.62%

Q 25 - Adam borrowed some money at the rate of 6% p.a. for the first two
years, at the rate of 9% p.a. for the next three years, and at the rate of 14%
p.a. for the period beyond five years. If he pays a total interest of Rs. 11,400 at
the end of nine years, how much money did he borrow?
A - 12,000
B - 13,000
C - 14,500
D - 12,500

Answer - A
Explanation
Let the sum borrowed be Z. Then,
(Zx6x2⁄100) + (Zx9x3⁄100) + (Zx14x4⁄100) = 11400
Therefore, 3Z⁄25 + 27Z⁄100 + 14Z⁄25) = 11400
? 95Z⁄100 = 11400
Z = (11400x100⁄95) = 12000

Q 26 - A certain sum of money amounts to Rs. 1008 in 2 years and to Rs. 1164
in 31⁄2 years. find the sum and the rate of interest?
A - Rs. 800, Rate 14%
B - Rs. 850, Rate 12%
C - Rs. 800, Rate 13%
D - Rs. 500, Rate 13%

Answer - C
Explanation
S.I. for 11⁄2years = Rs. (1164-1008)
=156
S.I. for 2 years = Rs. (156 x 2⁄3 x 2) = Rs. 208
Principal = Rs. (1008 - 208) = Rs. 800
Now, P = 800, T = 2, and S.I. = 208
Rate = (100x208⁄800x2)% = 13%
Q 27 - At what rate percent per annum will a sum of money double in 16
years?
A - 5%
B - 3%
C - 4%
D - 61⁄4%

Answer - D
Explanation
Let Principal = P, Then,
S.I. = P and T = 16 years
Rate = (100xP⁄Px16)%
= 6(1⁄4) %

Q 28 - The simple interest on a certain sum of money for 21⁄2 years at 12% per
annum is Rs. 40 less than the simple interest on the same sum for 31⁄2 years at
10% per annum. Find the sum?
A - 400
B - 800
C - 1600
D - 500
Answer - B
Explanation
Let the sum be Z then,
(Zx10x7⁄100x2) - (Zx12x5⁄100x2)
=40
7Z
⁄20 - 3Z⁄10 = 40
Z = 40 x 20
The sum is Rs. 800

Q 29 - A sum was put at simple interest at a certain rate for 3 years. Had it
been put at 2% higher rate, it would have fetched Rs. 360 more. Find the
sum?
A - 10000
B - 6000
C - 15000
D - 6500

Answer - B
Explanation
Let the sum be = P and original rate = R. Then,
(Px(R+2)x3⁄100) - (PxRx3⁄100) = 360
3PR + 6P - 3PR =36000
6P = 36000
P=6000
Q 30 - A person borrows Rs. 5000 for 2 years at 4% p.a. simple interest. he
immediately lends it to another person at 6 1⁄4% p.a. for 2 years find his gain in
the transaction per year?
A - 112
B - 112.50
C - 110
D - 100

Answer - B
Explanation
Gain in 2 years = Rs. [(5000x25x⁄42⁄100) - (500x4x2⁄100)]
= Rs. (625 - 400)
= Rs. 225
Gain in 1 year = Rs. (225⁄2)
= Rs. 112.50

Q 31 - How much time for an amount of Rs. 450 to yield Rs. 81 as interest at
4.5% per annum of simple interest?
A-4
B-5
C - 4.5
D-6

Answer - A
Explanation
Time = (100 x 81⁄450 x 4.5)
= 4 years
Q 32 - A sum of Rs. 12,500 amounts to Rs. 15,500 in 4 years at the rate of
simple interest. What is the rate of interest?
A-6
B - 6.5
C-5
D - 6.75
Answer - A
Explanation
S.I. = Rs. (15500 - 12500) = Rs. 3000
Rate = (100 x 3000⁄12500 x 4)
=6%

Q 33 - Reema took a loan of Rs. 1200 with simple interest for as many years as
the rate of interest. If she paid Rs. 432 as interest at the end of loan period,
what was the rate of interest?
A - 10
B-5
C-6
D-7
Answer - C
Explanation
Let Rate = R% and time also R years. Then,
(1200 x R x R⁄100) = 432
= 12R2 = 432
R2 = 36
R=6

Q 34 - A man took a loan from a bank at the rate of 12% p.a. simple interest.
After 3 years he had to pay Rs. 5400 interest only for the period. The principal
amount borrowed b him was?
A - 10000
B - 15000
C - 15500
D - 6500

Answer - B
Explanation
Principal = Rs. (100 x 5400⁄12 x 3)
= Rs. 15000

Q 35 - what is the present worth of Rs. 132 due in 2 years at 5% simple


interest per annum?
A - 120
B - 150
C - 155
D - 650
Answer - A
Explanation
Let the present worth be Rs. z then,
S.I. = Rs. (132 - z)
therefore (zx5x2⁄100) = 132 - z
10z = 13200 - 100z
110z = 13200
z = 120

Q 36 - A sum fetched a total simple interest of Rs. 4016.25 at the rate of 9


p.c.p.a in 5 years. What is the sum?
A - 10000
B - 8500
C - 9000
D - 8925

Answer - D
Explanation
Principal = (100 x 4016.25⁄9 x 5)
(401625⁄45)
= 8925
Q 37 - Rs. 800 becomes Rs. 956 in 3 years at a certain rate of simple interest. If
the rate of interest is increased by 4%, What amount will Rs. 800 become in 3
years?
A - 10000
B - 1025
C - 15500
D - 6500

Answer - B
Explanation
S.I. = (956 - 800) = 156
Rate = (100 x 156⁄800 x 3)
New Rate = (61⁄2 + 4)
= 10 1⁄2
New S.I. = Rs. (800 x 21 x⁄2 3⁄100)
=252
therefore New Amount = Rs. (800 + 252) = 1025

Q 38 - A certain amount earns simple interest of Rs. 1750 after 7 years. Had
the interest been 2 % more, how much more interest would it have earned?
A - 35
B - Data is inadequate
C - 245
D - 350

Answer - B
Explanation
We need to know the S.I., principal and time to find the rate.
Since the principal is not given, so the data is inadequate.

Q 39 - In how many years, Rs. 150 will produce the simple interest @ 8% as
Rs. 800 produce in 3 years @ 4(1⁄2?
A-6
B-8
C-9
D-8

Answer - C
Explanation
P = Rs. 800 R = 4 1⁄2 = (9⁄2 T = 3 years
S.I. = Rs. (800 x 9 x ⁄2 3⁄100) = 108 <
Now, P = Rs 150, S.I. = Rs. 108, R = 8%
Time = (100 x 108⁄150 x 8) = 9 years

Q 40 - A sum invested at 5% simple interest per annum grows to Rs. 504 in 4


years. The same amount at 10% simple interest per annum in 21⁄2 years will
grow to?
A - 420
B - 525
C - 450
D - 500
Answer - B
Explanation
Let the sum be Rs. z. Then,
S.I. = Rs. (504 - z)
therefore (z x 5 x 4⁄100) = 504 - z
20z = 50400 - 100z
120z = 50400
z = 420
Now P = 420, R = 10%, T = 5⁄2
S.I. = (420 x 10 x 5⁄100 x 2) = 105
Amount = Rs (420 + 105) = 525

Q 41 - what will be the ratio of simple interest earned by certain amount at the
same rate of interest for 6 years and that for 9 years?
A - 2:3
B - 1:4
C - 1:3
D - none

Answer - A
Explanation
Let the principal be P and rate of interest be R%
therefore Required Ratio =(PxRx6⁄100⁄PxRx9⁄100)
6PR
⁄9PR
=2:3
Q 42 - Nitin borrowed some money at 6% for the first three years, 9% for the
next 5 years and 13% for the period beyond 8 years. If the total interest paid
by him at the end of eleven years is Rs 8160, how much money did he
borrow?
A - 8000
B - 10000
C - 12000
D - Data inadequate

Answer - A
Explanation
Let the sum be Z. Then,
(Zx6x3⁄100) + (Zx9x5⁄100) + (Zx13x3⁄100) = 8160
= 18Z + 45Z + 39Z = (8160 x 100)
102Z = 816000
Z = 8000

Q 43 - An automobile financier claims to be lending money at simple interest,


but he includes the interest every six months for calculating the principal. If he
is changing an interest of 10%, the effective rate of interest becomes?
A - 12
B - 15
C - 10
D - 10.25
Answer - D
Explanation
Let the sum be Rs. 100. Then,
S.I. for first 6 months = Rs. (100 x 10 x 1⁄100 x 2) = Rs 5.
S.I. for last 6 months = Rs. (105 x 10 x 1⁄100 x 2) = Rs 5.25
So, amount at the end of 1 year = Rs. (100 + 5 + 5.25) = Rs. 110.25
Therefore Effective rate = (110.25 - 100) = 10.25%

Q 44 - A sum of money at simple interest amounts to RS. 815 in 3 years and to


854 in 4 years. The sum is?
A - 700
B - 690
C - 698
D - 650

Answer - C
Explanation
S.I. for 1 year = Rs. (854 - 815) = 39
S.I. for 3 years = Rs. (39 x 3) = 117
Therefore Principal = 815 - 117 = 698

Q 45 - An aggregate of cash adds up to Rs 5200 in 5 years and to Rs 5680 in 7


years at basic premium. The rate of interest per annum is:
A - 3%
B - 4%
C - 5%
D - 6%

Answer : D

Explanation

S.I for 2 years = Rs. (5680-5200) = Rs. 480


S.I for 1 year = Rs. (480/2) = 240
S.I for 5 year = (240*5) = 1200 Rs.
Principle = Rs. (5200-1200) = 4000.
Rate = (1200*100/4000*5) %p.a. = 6% p.a.

Q 22 - At what rate of simple interest a certain aggregate will be multiplied in


15 years?
A - 11/2% p.a.
B - 6% p.a.
C - 20/3% p.a
D - 15/2% p.a.

Answer : C

Explanation

Let the sum be rs. x. then, S.I = Rs.x.


Rate = (100*S.I / sum*time) = (100*x/x*15) % p.a = 20/3% p.a.
Q 46 - In what amount of time would the simple interest on a sure total be
0.125 times the vital at 10% for each annum?
A - 5/4 years
B - 7/4 years
C - 9/4 years
D - 11/4years

Answer : A

Explanation

Let the time be x years.


Let the sum be rs. P. Then, S.I = (0.125) P and R =10% p.a.
Time = (100*S.I/P*R) = {100*(0.125) P/ P*10} = 1.25 years = ( 5 / 4) Years

Q 47 - An aggregate of Rs 10000 is loaned mostly at 8% and the staying at 10%


p.a. On the off chance that the year?s interest on the normal is 9.2% the cash
loaned at 10% is:
A - Rs 6000
B - Rs 5500
C - Rs 5000
D - Rs 4500

Answer : A

Explanation

Ratio of investments = 0.8: 1.2: = 2:3


Money at 10% = Rs. (10000*3/5) = Rs. 6000
Q 48 - A man disperses Rs. 25300 among his three children A, B, C. In a
manner that the measure of there at 10% simple interest for a long time, 3
years and 4 years will be equivalent an offer is:
A - Rs. 7800
B - RS. 8400
C - RS. 9100
D - none of these

Answer : C

Explanation

Let the parts of A, B, C be x, Y, z respectively. Then


(x+ x*2*10/100)= (Y+ y*3*10/100) = (z+ z*4*10/100)
=> 6x/5 =13y/10 = 7z/5 =ℏ = 5ℏ/6, y = 10ℏ/13, z= 5ℏ/7
=> x: y: z = 5ℏ /6:10ℏ/13 :5ℏ/7 = 1/6:2/13: 1/7 = 91: 84: 78
A's part = Rs. (25300*91/253) = Rs. 9100.

Q 49 - A sum of Rs 1500 is lent out in two parts in such a way that the simple
interest on one part at 10% per annum for 5 years is equal to that on another
pert at 12.5% per annum for 4 years. The sum lent out at 12.5% is:
A - Rs 500
B - Rs 1000
C - Rs 750
D - Rs 1250

Answer : C
Explanation

Let the sum lent out at 12.5% be x


Sum lent out at 10% = 1500 - x

Now, (1500-x)*10*5/100 = x * 12.5*4/100


Or, 50(1500-x) = 50x
Or, 2x = 1500
Or, x = 750

Q 50 - The simple interest on a sum of money is 4/9 of the principal and the
number of years is equal to the rate percent per annum. The rate per annum
is
A - 5%
B - 20/3%
C - 6%
D - 7%

Answer : B

Explanation

We know, I = PTR/100
According to question,
I = 4P/9 and r = t
4P/9 = P*r*r/100
Or, r2 = 400/9
Or, r = 20/3
Q 51 - The prices of a T.V set worth Rs. 20000 is to be paid instalments of Rs.
1000 each. If the ROI be 6% per annum, and the first instalment be paid at the
time of purchase, then the value of the last instalment covering the interest as
well will be?
A - 1050
B - 2050
C - 3000
D - None of these

Answer : D

Explanation

Money paid in cash = Rs 1000


Balance payment = Rs (20000 - 1000) = Rs 19000

Q 52 - A sure entirety of cash adds up to rs. 756 in 2 years and to Rs. 873 in
7/2 years. Discover the aggregate and the rate of hobby.
A - Rs. 600,13% P.a
B - Rs. 600,12% P.a
C - Rs. 500,13% P.a
D - Rs. 500,12% P.a

Answer : A

Explanation

Sum in 7/2 years = Rs. 873, sum in 2 years= Rs. 756


S.I for 3/2 year = Rs. (873-756) = Rs. 117
S.I for a long time = Rs. (117*2/3*2) = rs. 156.
Standard = (sum in 2 year) - (s.I for 2 year) = Rs. (756-156)= Rs. 600
Presently P = 600 Rs. T= 2 years. What's more, S.I = 156 Rs.
∴ R = (100*S.I)/P*T = (100*156)/ (600*2) = 13% P.a.
Henceforth, aggregate = Rs. 600 and Rate = 13% P.a.
SPEED AND DISTANCE

Formulas

Speed = Distance⁄Time
Time = Distance⁄Speed
P km/hr = (P x 5⁄18)m/sec
P m/sec = (P x 18⁄5)km/hr

If the ratio of the speeds of P and Q is p:q, then the ratio of the times taken by
them to cover the same distance is

⁄ : ⁄ or q : p
1 p 1 q

The average speed of the journey is

(2pq⁄p+q)km/hr

if a man covers a certain distance at p km/hr and an equal distance at q km/hr

Q1. AC local trains travel at 74.2 km/hr. How much distance in kilometers will it
travel in 24 minutes?

A. 29.68 m

B. 2968 Km

C. 29680 m
D. 29.68 Km

Answer: D

Explanation:

Distance traveled in 1 min = (74.2/60) Km

Distance traveled in 24 min = (74.2/60)*24 = 29.68 Km

Q2. How many seconds does a rabbit take to cover a distance of 250 m, if the rabbit
runs at a speed of 8 km/hr?

A. 112.5 sec

B. 15/8 sec

C. 112.5 min

D. 555.5 sec

Answer: A

Explanation:

Rabbits Speed = 8 Km/hr = (5/18)* 8 = m/sec

∴ Time taken to cover 250 m = x250) sec = 112.5 sec

Q3. A cyclist covers a distance at a speed 26 Km/hr in 7 min 28 sec. What is the
distance in Km of the cyclist?

A. 9.10 Km
B. 3.23 Km

C. 4.32 Km

D. 11.64 Km

Answer: B

Explanation:

Distance = Speed x Time

Distance = 26 Km/hr X (7*60 + 28)/3600

Distance = 3.23 Km

Q4. A person jogged at a speed of 7 km/hr from point P to Q and came back from
point Q to P at the speed of 9 km/hr. What would be the ratio of the time taken by
the person jogging from point P to Q to that from point Q to P?

A. 7: 9

B. 7: 3

C. 9: 7

D. 3: 7

Answer: C

Explanation:

Ratio of speeds = 7 : 9

As the Distance traveled is the same, the ratio of Speed from P to Q to the Speed Q
to P will be the inverse of the Time taken in both cases.

∴ Ratio of times taken = 9 : 7


Q5. John covers one round of the city in 3406 sec by covering two-fourth of the
distance at 8 m/sec and the remaining at 10 m/sec. Find the total distance he
covered?

A. 34066.5 m

B. 10899.2 m

C. 30275.5 m

D. 41542.6 m

Answer: C

Explanation:

Let the total distance be x km.

Then, + = 3406

 = 3406

36x = 1089920

x = 30275.5 m

Q6. A Family traveled from Bombay to Goa at the speed of 80 kmph and came back
at the speed of 50 kmph. If the full journey took 7 hours 48 minutes, find the
distance from Bombay to Goa?

A. 220 Km

B.480 Km

C.240 Km

D.270 Km
Answer: C

Explanation:

Average speed = Km/hr = Km/hr= Km/hr

Distance Traveled in 7 hours 48 minutes, i.e., 7 hrs = ( x ) Km = 480 Km.

∴ Distance of Bombay to Goa = (480/2) = 240 Km

Q7. A car travels along the three sides of a triangle at the speeds of 40, 60 and 80
km/hr. find the average speed of the car in a triangular field?

A. 55.38 Km/hr.

B. 1040 Km/hr.

C. 38.55 Km/hr.

D. 41.62 Km/hr.

Answer: A

Explanation:

Let each side of the triangle be x km

And let the average speed of the plane around the field by y km/hr

Then, =

y = ( ) = 55.38
∴ Average speed = 55.38 Km/hr.

Q8. A Runner took a round of the four sides of a quadrilateral at the speeds of 12, 14,
18 and 20 km/hr. find the average speed of the runner in a quadrilateral field?

A. 15.36 Km/hr.

B. 64.57 Km/hr.

C. 17.5 Km/hr.

D. 16 Km/hr.

Answer: A

Explanation:

Let each side of the triangle be x km

And let the average speed of the plane around the field by y km/hr

Then, =

y = ( ) = 15.36

∴ Average speed = 15.36 Km/hr.

Q9. Rahul reaches his program 24 minutes after the program starts if his speed is 9
km/hr, and reaches 20 minutes before the start of program if his speed is 12 km/hr.
Find the distance where the program is from his starting point?

A. 34.03 km

B. 49.09 m
C. 26.4 Km

D. 79.2 m

Answer: C

Explanation:

Let the required distance be x Km

Difference in the times taken at two speeds = 44 min = (11/15) hr.

∴ =

12x -9x = 79.2

x = 26.4
Required distance is 26.4

Q10. Excluding the stoppages, the speed of a travel bus is 38 km/hr and including
the stoppages, the speed of the travel bus is 26 km/hr. How much time will it take
to cover the distance?

A. 1136.84 sec

B. 18.94 sec

C. 11.36 min

D. 15 min.

Answer: A

Explanation:

Due to stoppage, the bus covers (38 – 26) = 12 km less per hour.
Time taken to cover 12 km = 60 * 60 = 1136.84 sec

Q11. Dombivli and Kopar are two stations 460 km apart. The U train starts from
Dombivli at 9 a.m. and travels towards Kopar at 55 kmph. Another train starts from
kopar at 10 a.m. and travels towards dombivli at 45 kmph. At what time do they
meet each other?

A. 2:03 p.m

B. 1:13 p.m

C. 3:09 p.m

D. 11.08 Km/hr

Answer: A

Explanation:

Suppose they meet x hours after 9 a.m. Then,

(Distance moved by first in x hrs) + [Distance moved by second in (x – 1) hrs] = 460

∴ 55x + 45(x - 1) = 460

100x = 505

x = 5.05

So they meet 5 hrs. 3 min after 9 am, i.e at 2.03 p.m

Q12. A local train leaves Thane station at a certain time and at a fixed speed. After
240 sec, a luxury train leaves the same station and moves in the same direction at a
uniform speed of 22 m/s. This train catches up to the local train in 120 sec. Find the
speed of the local train?
A. 660 m/s

B. 11.54 m/s

C. 7.34 m/s

D. 15.62 m/s

Answer: C

Explanation:

Let the speed of the local train be x m/s

Distance covered by local train in 360 sec = Distance covered by luxury train in
120sec.

∴ 360x =120 * 22

x = 22/3 m/s

So, speed of local train = (22/3) m/s =7.33 m/s

Q13. A vehicle speed of 80.6 km/hr will be similar to?

A. 22.39 m/sec

B. 18.46 m/sec

C. 29.00 m/sec

D. 15.64 m/sec

Answer: A

Explanation:

80.6 km/hr = (5/18)*80.6 = 22.39 m/s


Q14. A small girl was riding her tricycle covering 144 meters in 36 seconds. What is
her speed in km per hour?

A. 14.4

B. 4

C. 12.1

D. 15

Answer: A

Explanation:

Speed = m/s = 4 m/s

= = 14.4 Kmph

Q15. A minivan went by 1862 Km at a speed of 76 Kmph. What is the time in minutes
required?

A. 1470

B. 24.5

C. 2448

D. 88200

Answer: A

Explanation:

Time = = = 24.5 hrs


= 24.5 * 60 = 1470 minutes

Q16. R is running with the speed 18 m/s on a track while S is running with a speed of
78 kmph. What will be the difference in speed of R and S in m/s?

A. 3.66

B. 21.66

C. 60

D. 22

Answer: A

Explanation:

R speed = 18 m/s

S speed = 78 Kmph = (5/18)*78 = 21.66 m/sec

Difference = (21.66-18) = 3.66 m/s

Q17. Suresh can reach taj mahal palace in 42 hours. If he decreases speed by 1/13
times, he travels 18 km less in that time. What is the speed of suresh?

A. 5.57

B. 3.23

C. 4.27

D. 6.36

Answer: A
Explanation:

Let the speed be x km/hr

Then, 42x - (42x ) = 18

3.23x =18

x =5.57 km/hr.

Q18. A motorcyclist rides a bike at 24 kmph covering 64 km and further 86 km at 32


kmph. Find the average speed during the rides?

A. 28.01 Km/hr.

B. 26.14 Km/hr.

C. 21.24 Km/hr.

D. 22.63 Km/hr.

Answer: A

Explanation:

Total distance traveled = (64 + 86) km = 150 km.

Total time taken = hr = hr

∴ Average speed = 150 = 28.01 km/hr

Q19. An auto rickshaw travels at a speed of 32.4 km/hr. How much time in minutes
will it require to cover 12 km distances?

A. 0.3703
B. 1333.33

C. 37.03

D. 22.22

Answer: D

Explanation:

Time = = = 0.3703 hrs

=0.3703 * 60 = 22.22 minutes

Q20. A shoplifter is spotted in the mall by a manager from a distance of 25 meters.


When the manager starts chasing, the shoplifter also starts getting faster. If the
speed of the shoplifter is 5 km/hr and the manager 7 km/hr, how far will the
shoplifter have to run before he gets caught?

A. 52.5

B. 65

C. 42.5

D. 62.5

Answer: D

Explanation:

Relative speed of the policeman = (7 – 5) km/hr = 2 km/hr.

Time taken by manager to cover 25 m = hr = hr

In 1/80 hr, the thief covers a distance = 5= km = 62.5 m


Q21. The ratio of the speeds of a bike, electric bike and car is 6 : 13 : 5. The average
speed of the bike, electric bike and car is 64 km/hr. What is the average speed of a
bike, electric bike together?

A. 48

B. 104

C. 64

D. 76

Answer: D

Explanation:

Let the speeds of the bike, electric bike and car be 6x, 12x and 5x km/hr
respectively.

Then, = 64

8x = 64 x = 8.

Speed of the bike = 48 km/hr ;

Speed of the electric bike = 104 km/hr.

Average speed of bike and electric bike

= = 76 km/hr

Q22. Which of the following Mahindra cars is the fastest?

A. 30 m/sec

B.1800 m/min
C. 108 km/hr.

D. None of these

Answer: D

Explanation:

30 m/s = 30 = 108 km/hr.

30 m/s = 30 * 60 = 1800 m/min

So, all having same speed

None of these is the answer

Q23. Sachin takes 24 minutes to one round of track at a speed of 5 km/hr. If he


starts jogging at a speed of 8 km/hr. In what time will he complete the same round
of track?

A. 18

B.20

C. 2

D. 15

Answer: D

Explanation:

Distance = Speed x Time = 5 = 2 km.

Required time = = = hr = 15 minutes


Q24. Including the stoppages, the speed of a travel carriage is 26 km/hr and
excluding the stoppages, the speed of the travel carriage is 30 km/hr. How much
time bus will take to cover stopover time?

A. 8 min

B. 16 min

C. 0.13 min

D. 25 min

Answer: A

Explanation:

Due to stoppage, the car covers (30 – 26) = 4 km less per hour.

Time taken to cover 4 km = 60 = 8 min

Q25. A person is walking at a speed of ‘A’ kmph to cover a distance of 2 km. But, due
to the undulating surface, his speed got reduced by ‘B’ kmph (A > B). If he takes T
hours to cover the distance, then?

A. A – B =

B.A + B =

C. B -A =

D. A – T =

Answer: D

Explanation:
Required Speed = = A–B=

Q26. Sound is said to travel in air at about 2200 feet per second. A girl hears the ax
striking the tree, 12/8 seconds after she sees it strike the tree. How far is the girl
from the wood chopper?

A. 2200

B.4400

C. 5500

D. 3300

Answer: D

Explanation:

Distance = Speed x Time = 2200 = 3300 feet.

Q27. A ball was hit by batman’s it traveled 28m at 4 m/sec speed. What is the time
required to travel this distance?

A. 7

B. 9

C. 3

D. 4

Answer: A

Explanation:

Time = = = 7 sec
Q 28 - What is meters/sec for 54 km/hr?
A - 15 m/sec
B - 20 m/sec
C - 25 m/sec
D - 30 m/sec

Answer - A
Explanation

54 km/hr = (54*5/18) m/sec = 15 m/sec.

Q 29 - What is km/hr for 16 m/sec?


A - 53.6km/hr
B - 55.6km/hr
C - 57.6km/hr
D - 59.6km/hr

Answer - C
Explanation

16 m/sec = (16*18/5)km/hr = 288/5km/hr =57.6 km/hr.


Q 30 - Anita can cover a sure separation in 1 hr 24min by covering two-third of
the separation at 4km/hr and the rest at5 km/hr. Find the aggregate
separation.
A - 3 kms
B - 4 kms
C - 5 kms
D - 6 kms

Answer - D
Explanation

Let the aggregate separation be x km.


Then, 2/3x/4+1/3x/5=7/5
=> x/6+x/15=7/5
=> 5x+2x=42
=> 7x=42
=> x=6.
∴ Total separation = 6 kms

Q 31 - A man strolls from his home to the railroad station. On the off chance
that he strolls at 5 km/hr, he misses a train by 7 minutes. However, on the off
chance that he strolls at 6km/hr, he achieves the station 5 minute before the
flight of the train. Discover the separation secured by him to achieve the
station.
A - 5 kms
B - 6 kms
C - 7 kms
D - 8 kms
Answer - B
Explanation

Let the required separation be x km. At that point,


X/5 - x/6 = 12/60 (distinction between two time interims is 12 min.)
=> x/5 - x/6 = 1/5
=> 6x-5x=6
=> x= 6
Required separation = 6 kms

Q 32 - Strolling at 7/8 of its typical velocity, a train is 10 minutes past the point
of no return. Locate its standard time to cover the trip?
A - 60 min
B - 70 min
C - 80 min
D - 90 min

Answer - B
Explanation

New speed = 7/8 of its standard velocity


New time taken = 8/7 of the standard time.
(8/7 of the standard time)- (common time) = 10 min.
=> 1/7 of the standard time = 10 min
=> usual time = 70 min.
Q 33 - Hitesh covers a sure separation via auto driving at 70 km/hr and returns
back to the beginning stage riding on a bike at 55km/hr. locate his normal
velocity for the entire trip?
A - 62.6 km/hr
B - 61.6 km/hr
C - 60.6 km/hr
D - 59.6 km/hr

Answer - B
Explanation

Normal velocity = 2xy/(x+y) km/hr = (2*70*55)/ (70+55) km/hr


= (2*70*55)/125 km/hr = 308/5 km/hr = 61.6 km/hr

Q 34 - The separation between two stations A and B is 450 km. A train begins
at 4 pm from A and moves towards B at a normal velocity of 60 km/hr.
Another train begins from B at 3.20 p.m and moves towards A at a normal
velocity of 80 km/hr. How a long way from A will the two train s meet and
what time?
A - 6:50 pm
B - 5:50 pm
C - 4:50 pm
D - 3:50 pm

Answer - A
Explanation

Assume two trains meet at x km from A


(time taken by B to cover (450-x) km-(time taken by A to cover x km) = 40/60
=> (450-x)/80 - x/60 = 40/60 ?3 (450-x) - 4x = 160
=> 7x=1190 ∴ x = 170
In this way the two trains meet at a separation of 170 km from A.
Time taken by A to cover 170 km = 170/60 = 2hrs 50 min.
Along these lines, the two trains meet at 6:50 pm

Q 35 - A man cycles from A to B, a separation of 21 km in 1 hr 40 min. The


street from A is level for 13 km and afterward it is tough to B. The man's
normal rate on level is 15 km/hr. Locate his normal tough pace?
A - 10 km/hr
B - 11 km/hr
C - 12 km/hr
D - 13 km/hr

Answer - A
Explanation

Let the normal tough rate be x km/hr. at that point,


13/15 + 8/x = 5/3
=>8/x= (5/3-13/15) = 12/15 = 4/5
=>x = (8*5)/4 = 10
∴ Normal tough rate = 10 km/hr

Q 36 - A hoodlum is spotted by a policeman from a separation of 100 meters.


At the point when the policeman begins the pursuit, the criminal likewise
begin s running. In the event that the pace of the criminal be 8km/hr and that
of the policeman 10 km/hr, how far the hoodlum will have keep running
before he is overwhelmed?
A - 200 m
B - 300 m
C - 400 m
D - 500 m

Answer - C
Explanation

Relative pace of the policeman = (10-8) km/hr = 2 km/hr


Time taken by policeman to cover 100 m = (100/1000*1/2) hr = 1/20 hr
In 1/20 hr, the cheat covers a separation of (8*1/20) km= (2/5 km) =
(2/5*1000) m = 400 m

Q 37 - I walk a sure separation and ride back setting aside an aggregate time
of 37 minutes. I could walk both routes in 55 minutes. To what extent would it
take me to ride both ways?
A - 18 min
B - 19 min
C - 20 min
D - 21 min
Answer - B
Explanation

Let the given separation be x km. At that point,


(Time taken to walk X km)+ (time taken to ride x km) = 37 min.
=> (time taken to walk 2X km) + (time taken to ride 2x km) = 74 min.
=> 55 min + (time taken to ride 2 x km) = 74
=> time taken to ride 2x km = 19 min.
Q 38 - A and B are two stations 390 km separated. A train begins from An at
10 am and ventures towards B at 65 kmph. Another train begins from B at 11
am and towards A at 35 kmph. AT what time they meet?
A - 2.15 pm
B - 3.15 pm
C - 4.15 pm
D - 5.15 pm

Answer - A
Explanation

Assume they meet x hours after 10 am then,


(separation moved by first in x hrs.) + (separation moved by second in (x-1) hrs)
= 390
∴ 65x+ 35(x-1) = 390
=> 100x =425
=> x= 17/4
In this way, they meet 4 hrs 15 min. after 10 am at 2.15 pm

Q 39 - A products train leaves a station at a sure time and at a settled rate.


Following 6 hours, an express prepare leaves the same station and moves in
the same bearing at a uniform velocity of 90 kmph. This train makes up for
lost time the merchandise train in 4 hours. Discover the velocity of the
merchandise train.
A - 33 kmph
B - 34 kmph
C - 35 kmph
D - 36 kmph
Answer - B
Explanation

Let the velocity of the merchandise train be x kmph.


Separation secured by products train in 10 hours = separation secured by
express prepare in 4 hours
∴ 10x =4*90 or x = 36
Along these lines, rate of products train = 36 kmph

Q 40 - A man finishes 30 km of a voyage at 6km/hr and the staying 40km of


the venture in 5 hr.His normal pace for the entire voyage is:
A - 70/11 km/hr
B - 7 km/hr
C - 15/2 km
D - 8 km/hr

Answer : B

Explanation

Total distance = (30+40)km= 70 km


Total time taken = (30/6 +5) hrs =10 hrs
Average speed = 70/10 km/hr = 7 km/hr

Q 41 - A is twice as quick as B and B is thrice as quick as C. The excursion


secured by C in 42 min. will be secured by A in
A - 7 min.
B - 14 min.
C - 28 min.
D - 63 min.

Answer : A

Explanation

Let c speed be x meters/min.


Then, B speed=3x meters /min and A speed =6x meters/ min.
Ratio of speed of A and C =ratio of times taken by C and A
6x:x=42:ymin⇒6x/x=42/y⇒y=42/6min=7 min.

Q 42 - If an understudy strolls from his home to class at 5km/hrs, he is late by


30 min. However, on the off chance that he strolls at 6 km/hr. he is late by 5
min. just. The separation of his school from his home is:
A - 2.5 km
B - 3.6 km
C - 5.5 km
D - 12.5 km

Answer : D

Explanation

Let the required distance be x km. then,


x/5 - x/6 = 25/7 (difference between two times is 25 min.)
⇒ 12x- 10 x = 25 ⇒2x = 25 ⇒ x= 25/2 km = 12.5 km

Q 43 - A train secured a separation at a uniform velocity. On the off chance


that the train had been 6 km/hr speedier, it would have taken 4 hours not
exactly the booked time, and if the train were slower by 6 km/hr, the train
would have taken 6 hours more than the planned time. The length of the trip
is?
A - 700 km
B - 720 km
C - 740 km
D - 760 km

Answer : B

Explanation

Let the required distance be x km and uniform speed by y km/hr


x/y - x/(y+6) = 4 ...(a) x/(y-6) - x/y = 6 ...(b)
⇒xy+6x-xy = 4y (y+6) and xy - xy +6x = 6y (y-6)
⇒4y2+24 y - 6x = 0 and 6y2- 36y - 6x = 0
⇒2y2- 60 y = 0
⇒ 2y (y-30) = 0 ⇒y = 30
∴x/30 - x/36 = 4 ⇒6x- 5x = 720 ⇒x = 720 km

Q 44 - A star is 8.1* 10ⁱ3km far from the earth. Assume light goes at the pace
of 3.0* 10⁵ km for every second. To what extent will it take light from star to
achieve the earth?
A - 7.5 * 103 hrs
B - 7.5 * 10⁴ hrs
C - 2.7 * 10ⁱ⁰ sec
D - 2.7 * 10ⁱⁱ sec

Answer : B
Explanation

(3*10⁵) km is covered in 1 sec.


(8.1 * 10ⁱ3) km is covered in (1/3 *10⁵* 8.1*10ⁱ3) sec = (2.7 *10⁸*1/60*1/60) hrs
= (2.7*10⁶)/36 hrs= (2.7 *100*10⁴)/36 hrs = (7.5 *10⁴) hrs.

Q 45 - A constable is 114 m behind a thief. The constable runs 21 m and the


thief15 m in a moment. In what the reality of the situation will become
obvious eventually constable catch the criminal?
A - 16 min
B - 17 min
C - 18 min
D - 19 min

Answer : D

Explanation

(21-15) m i.e.6m is covered in 1 min.


114m will be covered in (1/6*114) min=19 min.

Q 46 - Renu began cycling along the limits of a square field ABCD from corner
point A. after thirty minutes, he came to the corner point C, slantingly inverse
to A. In the event that his rate was 8 km/hr, the zone of the field is:
A - 64 sq. km
B - 8 sq. km
C - 4 sq. km
D - cannot be resolved
Answer : B

Explanation

Length of diagonal = (8*1/2 )km = 4 km


Area of the field = (1/2 *4*4) sq. km = 8 sq. km

Q 47 - The proportion between the rates of strolling of A and B is 2:3. In the


event that the time taken by B to cover a sure separation is 36 minutes, the
time taken by A to cover that much separation is
A - 24 min
B - 54 min
C - 48 min
D - none of these

Answer : B

Explanation

Ratio of time taken = 1/2:1/3 = 3:2


Time taken by B = 36 min. let the time taken by A be x min.
∴x/36 = 3/2 ⇒x = (3*36/2) min. = 54 min

Q 48 - The proportion between the rates of going of An and B is 2:3 and in this
manner A takes 10 minute more than the time taken by B to achieve a
destination. In the event that A had strolled at twofold speed, he would have
secured the separation in
A - 30 min
B - 25 min
C - 20 min
D - 15 min

Answer : D

Explanation

Ratio of time taken by A and B = 1/2: 1/3


Suppose B takes x min. Then, A takes (x+10) min.
∴(x+10): x= 1/2: 1/3 = 3:2 ⇒ x+10/x = 3/2 ⇒ 2x+20 = 3x ⇒x = 20
Thus B takes 20 min. and A takes 30 min.
AT double speed A would covers it in 15 min.

Q 49 - A agriculturist voyaged a separation of 61 km in 9 hours. He voyaged


halfway by walking at 4 km/hr and incompletely on bike at 9 km/hr. The
separation went by walking is:
A - 14 km
B - 15 km
C - 16 km
D - 17 km

Answer : C

Explanation

Let the distance travelled on foot be x km


Then, distance covered on bicycle = (61-x) km
∴x/4 + 61-x/9 =9 ⇒ 9x+4(61-x)= 324 ⇒ 5x = (324-244)= 80 ⇒x = 16
Distance covered on foot = 16 km
AGES

Solving problems on ages is a two step process.

1. Create linear equations based on statements provided.

2. Solve the equations to get the values as required ages.

Q1. Niraj was the father of two children named Rohan and Mohan whose ratio of
ages was 6 : 8 respectively. After 12 year, the ratio was 9 : 10 respectively. What is
the age of Rohan now?

A.9 years

B.6 years

C. 12 years

D.18 years

Answer: B

Explanation:

Let Rohan’s age be 6x years. Then, Mohan’s age = 8x years

∴ =

⇒ 60x + 120 = 72x + 108

⇒ 12x = 12

⇒x=1

∴ Present age of Rohan is ( 6 x 1) = 6 years


Q2. The present ratio of age of A and B is given as 6 : 8. The age difference between
B' present age and A's age hence 12 years is 4. What is the difference between their
age?

A. 16

B. 10

C. 24

D. 12

Answer: A

Explanation:

Let age be A and B.

⇒A : B = 6 : 8 ⇒ 6x , 8x

⇒Q - (P +12) = 4

⇒8x - ( 6x + 12 ) = 4

⇒2x = 16

⇒X = 8

∴ Difference of their age = Q - P = 64 - 48 = 16

Q3. The sum of the ages of Farhan present age and Rafi 6 years ago is 14. The
present ratio of their age is 4 : 6. What is the product of their ages?

A. 68

B. 118
C. 72

D. 96

Answer: D

Explanation:

Let Fahran age be P and Rafi age be Q.

⇒P : Q = : 8 ⇒ 4x , 6x

⇒Q + (P - 6) = 14

⇒6x + ( 4x - 6 ) = 14

⇒10x = 20

⇒X = 2

∴ Product of their age = Q * P = 12* 8 = 96

Q4. In a family the ratio between the present ages of father and son is 8: 6
respectively. The ratio between father's age 6 years ago and son's 6 year hence is 2
: 2. What is the ratio between father's age 6 years hence and son's age 6 years ago ?

A. 9 : 5

B. 8 : 3

C. 6 : 4

D. 3 : 2

Answer: A

Explanation:
Let father be A and Son be B.

A : B = 8 : 6 ⇒ 8x : 6x

8x - 6 : 6x + 6 = 2 : 2

8x - 6 = 6x + 6

⇒ 2x = 12 ⇒ x = 6

So father age = 8*6 = 48 and son age = 6* 6 = 36

Father age 6 years hence = 48 + 6 = 54

Son age 6 years ago = 36 - 6 = 30

Ratio = 54 : 30 = 9 : 5

Q5. In the group of friends the ratio between the present ages of one friend over
another is 6: 4 respectively. The ratio between the 1st friend's age is 3 years ago and
the 2nd 3 year hence is 3 : 3. What is the ratio between the 1st friend's age 3 years
hence and 2nd friend's age 3 years ago ?

A. 7 : 3

B. 8 : 6

C. 6 : 4

D. 3 : 2

Answer: A

Explanation:

Let 1st friend be A and 2nd be B.

A : B = 6 : 4 ⇒ 6x : 4x

6x - 3 : 4x + 3 = 3 : 3

(6x - 3) = (4x + 3)

⇒ 2x = 6 ⇒ x = 3
So 1st friend’s age = 6*3 = 18 and 2nd friend’s age = 4* 3 = 12

Father age 3 years hence = 18 + 3 = 21

Son age 3 years ago = 18 - 3 = 15

Ratio = 21 : 15 = 7 : 3

Q6. Kushi’s age is 6 years more than three times Prachi's age. Sakshi's age is 16
years less than 16 times Prachi's age. If Sakshi is 4 times as old as Kushi’s age,
what is the age of khushi?

A.36

B. 16

C. 10

D. 14

Answer: A

Explanation:

Kushi = 3 Prachi + 6 ………i

Sakshi = 16 Prachi - 16…….ii

Sakshi = 4 kushi…….iii

Substitute iii in ii

4 ( Kushi ) = 16 Prachi - 16

4 ( 3 Prachi + 6 ) = 16 prachi - 16

12 prachi + 24 = 16 prachi - 16

4 prachi = 40

Prachi = 10
So, Kushi = 3 x10 + 6 = 30 +6 = 36

Q7. Krishna’s daughter's age is 1/5 of Krishna’s wife's age. Krishna’s wife’s age is
5/6 of Krishna's age and Krishna’s age is 4/6 of Krishna’s uncle’s age. Find the age
of Krishna’s daughter, if Krishna’s uncle is 45 years old.

A. 15

B. 20

C. 10

D. 5

Answer: D

Explanation:

Krishna’s Uncle’s age = 45 years, Krishna’s age

= x 45) years = 30 years

Krishna’s wife's age = x 30) years = 25 years

Krishna’s daughter's age = x 25) years = 5 years

Q8. D’s age 6 years ago was 4 times the present age of E. At present, F’s age is 3
times the age of E. Also, F is 16 years younger than D. What is the present age of F?

A. 16

B. 36

C. 30
D. 10

Answer: C

Explanation:

Let the present age of E be a year.

6 years ago D’s age = 4a years

Then, present age of D is (4a + 6)

F’s present age = 3a

According to the given information

Now, (4a + 6) – 3a = 16 ⇒ a = 10 year

∴ Present age of F = 3a = 3 × 10 = 30 years

Q9. Ritesh’s age is 1/4 th of hisfather’s age. Ritesh’s father’s age will be 2 times
of Shara’s age after 8 years. If Shara’s 12th birthday was celebrated 4 years ago,
then what is Ritesh’s present age?

A. 14

B. 12

C. 16

D. 18

Answer: B

Explanation:

Shara’s present age = (12 + 4) years = 16 years.


Ritesh’s Father’s = 2 (16 + 8) years = 48 years.

Then, present age of Ritesh years = years = 12 years

Q10. In a family ratio between the ages of A,B & C is 3 : 4 : 5. If the difference
between the ages of B and C is 5 years, then what is the difference between the
ages of A and C?

A. 14

B. 10

C. 16

D. 18

Answer: B

Explanation:

Let A’s age be 3x years.

Then, B’s age = 4x years and C's age = 5x years.

∴ 5x – 4x = 5 ⇒ x = 5.

Required difference = (5x – 3x) years = 2x years = (2 × 5) years = 10 years.

Q11. Twelve years ago, a Ship was six times as old as a new Ship. 2 years hence,
Four times Ship's age will be equal to three times the age of the new Ship. What is
the present age of the old Ship?

A. 14

B. 13

C. 16

D. 15
Answer: B

Explanation:

Let ship’s age 12 years ago be x years.

Then, new ship’s age 12 years ago = 6x years.

Old ship’s present age = (x + 12) years,

New ship’s present age= (6x + 12) years.

∴ 3 [(6x + 12) + 2] = 4 [(x + 12) + 2]

⇒ 3 (6x + 14) = 4 (x + 14)

⇒ 18x + 42 = 4x + 56

⇒ 14x = 14

⇒ x = 1.

∴ old ship’s present age = (1 + 12) years = 13 year

Q12. The sum of the present ages of Radha and Krishna is 60 years. Six years hence,
the ratio of their ages was 4 : 5 respectively. What is Krishna’s present age?

A. 72

B.34

C.56

D. 54

Answer: B

Explanation:
Let the ages of Radha and Krishna 6 years hence be 4x years and 5x years
respectively. Then, Radha’s present age = (4x - 6) years,

Krishna’s present age = (5x - 6) years.

∴ (5x - 6) + (4x - 6) = 60

⇒ 9x = 72

⇒ x = 8.

Krishna’s present age = (8 × 5-6) years = 34 years.

Q 13 - If the ratio of ages of two persons Ram and sham is 5:4 . After Three
years their age ratio changes and becomes 11:9. In that case tell about the
present age of mr. sham.

A - 23 years

B - 24 years

C - 25 years

D - 26 years

Answer - B

Explanation

If the age of Mr. Ram 5x and 4x is the age of Mr. sham.


Then, ( 5x+3) /( 4x+3) = 11/9 ⇒ 9 (5x+3)= 11 (4x+3) ⇒ x= (33-27) = 6
So the present age of Mr. sham = 6*4 = 24 years.
Q 14 - A mother is 30 time older in the comparison of her daughter. After the
period of 18 year , the mother age would be thrice in the comparison of his
daughter . In that case tell about the present age of mother.

A - 40 years

B - 41 years

C - 42 years

D - 43 years

Answer - A

Explanation

Let daughter present age be x year .


in that case mother present age would be = 30x years
30x+ 18 = 3 (x+ 18) ⇒ 27x = 36 ⇒ x = 4/3
∴ so the present age of mother = (30* 4/3) = 40 years.

Q 15 - The ratio of present ages of three persons ajay , vijay and sanjay are in
the proportion of 4: 7: 9. Before 8 year total sum of their age is 56. What
should be the present ages?

A - 28 and 36 years

B - 28 and 38 years

C - 30 and 36 years

D - 36 and 28 years
Answer - A

Explanation

If the present age of ajay , vijay and sanjay is 4x , 7x and 9x years.


Total sum of ages of ajay, vijay and sanjay before 8 years ago = (4x-8)+(7x-
8)+(9x-8)
=(20x-24) years.
∴ 20x-24 = 56 ⇒ 20x = 80 ⇒ x= 4
Hence, it proves that age of ajay is 4*4 = 16 years,
vijay (7*4) = 28 years and sanjay (9* 4) = 36 years.

Q 16 - Daughter's present age is 2/5 in the comparison of her mother .8 year


later , age of her daughter will be 1/2 in the comparison of her mother. Find
out mother present age?

A - 39 years.

B - 40 years

C - 41 years

D - 42 years

Answer - B

Explanation

If the present age of mother is equal to x year.


In that situation the daughter present age would be = 2x/5 years.
2x/5 + 8 = 1/2 (x+8) ⇒ 4x+ 80 = 5x+40 ⇒ x = 40.
The mother age at the present time is = 40 years.

Q 17 - Ajay age was double in the comparison of bhuvan before 3 years. Seven
years hence, the sum of both ages would be 83 years. What should be the age
at the present time of both?

A - 43 years

B - 44 years

C - 45 years

D - 46 years

Answer - C

Explanation

Before 3 year let bhuvan age be x years.


3 years before , ajay age will be 2x years.
Now Bhuwan's age =(x+3) years and ajay age = (2x+3) years.
(x+3)+7+(2x+3)+7 = 83 ⇒ 3x+20 = 83 ⇒ 3x = 63 ⇒ x = 21
Now the bhuwan present age = (21+3) = 24 years
Now the ajay present age = (2 *21+3) years = 45 years.
Q 18 - I am 4 year older in the comparison of my sister, but my brother who is
the youngest among us is 7 year younger to myself. My father is three times in
the comparison of my brother. The present age of my sister 18 year and my
father is 3 year older in the comparison of my mother. In that situation what
should be the present age of my mother?.

A - 42 years

B - 43 years

C - 44 years

D - 45 years

Answer - A

Explanation

If my sister age is x years. Then,


Sister - x
I - x+4
Brother - (x+4-7) = x-3
Father - 3 (x-3)
Given x = 18
∴ Father's age = 3(18-3) = 45 years.
Mother age = (45-3) = 42 years.
Q 19 - Ajay is as much younger to vijay as he is older to vinay. If 48 years is the
sum of the ages of vijay and buwan . Then find out the present age of Mr. ajay
?

A - 21 years

B - 22 years

C - 23 years

D - 24 years

Answer - D

Explanation

V-A = A- B ⇒ V+B = 2A =48 ⇒ 24


Now, We can say that the present age of Mr. Ajay is 24 years.

Q 20 - If 100 year is equal to the sum of the ages of father and son. 2:1 was
the ratio of father and son before the period of 5 years. Find out the ratio of
ages which would be after the period 10 year.

A - 3:4

B - 3:5

C - 4:3

D - 5:3

Answer - D
Explanation

If the age of father at the present time = x years


His son age at the present time = (100-x) years.
x-5 / (100-x-5) = 2/1 ⇒ (x-5) = 2(95-x) ⇒ 3x = 195 ⇒ x = 65
Ratio of the ages of man and son after 10 years = (65+10)/(35+10)= 75/45 =
5/3 = 5:3

Q.21 - A father said his son , " I was as old as you are at present at the time of
your birth. " If the father age is 38 now, the son age 5 years back was :
A. 14
B. 19
C. 33
D. 38

Answer - A

Explanation

Let the son's present age be x years .Then, (38 - x) = x => x= 19.

Son's age 5 years back = (19 - 5) = 14 years

Q.22 - The total age of A and B is 12 years more than the total age of B and C. C
is how many years younger than A ?
A. 12
B. 13
C. 14
D. 15

Answer - A

Explanation

(A+B) - (B+C) = 12

A - C = 12.

C is younger than A by 12 years

Q 23 - The ages of Arzoo and Arnav are in the ratio of 11:13 respectively. After
7 years the ratio of their ages will be 20:23. What is the difference in years
between their ages?
A - 4 years
B - 7 years
C - 6 years
D - 5 years

Answer : C

Explanation

Let the present ages of Arzoo and Arnav be 11x and 13x respectively.
According to the question,
(11x + 7) / (13x + 7) = 20/23
Or, 26x + 140 = 253x + 161
Or, 260x - 253x = 161 - 140
Or, 7x = 21
Or, x = 3

Difference between their ages = 13x - 11x


= 2x
= 2*3
= 6 years

Q 24 - In a family the average age of the father and mother is 38 years


whereas the average age of father, mother and their only daughter is 28
years. Then the age of the daughter is -
A - 5 years
B - 6 years
C - 7 years
D - 8 years

Answer : D

Explanation

The age of the daughter = (3*28) - (2*38)


= 84 - 76
= 8 years

Q 25 - The ratio of the present ages of a son and his father is 1:5 and that of
his mother and father is 4: 5. After 2 years the ratio of the ages of the son to
that of his mother becomes 3 : 10. What is the present age of the father?
A - 37 years
B - 34 years
C - 30 years
D - 35 years

Answer : D

Explanation

Let the present age of the son be x years.


present age of the father = 5x years and that of the mother = 4x years.

2 years hence, age of son = (x + 2) years


And age of the will be = (4x + 2) years

As per the question,


(x + 2)/(4x + 2) = 3/10
Or, 10 (x+2) = 3 (4x + 2)
Or, 10x + 20 = 12x + 6
Or, 2x = 14
Or, x = 7
Present age of father = 5x = 5*7 = 35 years

Q 26 - A man five times older in the comparison of his son in the present time
but before the period of 5 year the father age in the comparison of his son
was 10 times older . Find out present age of the father?
A - 45 year
B - 48 years
C - 49 years
D - 52 years

Answer : A

Explanation

Let the present age of the son be x year. Then , present age of the man = 5x
year
∴ (5x-5 = 10 (x-5) ⇒ 5x= 45 ⇒ x = 9
Age at the present time of the father is = (5*9) = 45 year

Q 27 - If 100 year is equal to the sum of the ages of father and son. 2:1 was
the ratio of father and son before the period of 5 years . Find out the ratio of
ages which would be after the period 10 year.
A - 4:3
B - 5:3
C - 3:5
D - 10:7
Answer : B

Explanation

If x is the father age ,then son age will be (100-x)


x-5/ (100-x)-5 =2/1 ⇒ (x-5)= 2(95-x) ⇒ x =-5= 190 -2x ⇒ 3x =195 ⇒ x =65
father age = 65 yrs. And son age = 100-65 = 35 year
required ratio = (65+10) / (35+10) =75/45 = 5:3

Q 28 - What should be the present age of two person if 5:7 is the present age
ratio of two person and 3:5 was the age ratio before the period of 16 year?
A - 30, 44 years
B - 35, 52 years
C - 40, 56 years
D - 45, 60 years

Answer : C

Explanation

If the present age of two persons are 5x years and 7x years.


5x-16/ 7x-16 =3/5 ⇒ 5(5x-16) =3(7x-16)
⇒ 4x= (80-48) = 32 ⇒ x =8
Hence, their present ages are 40 years and 56 years.

Q 29 - A mother have 7 times in the comparison of her daughter at the time of


1 year ago . At the present time mother age is equal to the square of her son .
What should be the time age of mother?
A - 7 years
B - 36 years
C - 49 years
D - 64 years

Answer : B

Explanation

Let the present age of the son be x years.


Man present age =7(x-1)+1 years = (7x-6 ) yrs.
7x-6 = x2 ⇒ x2-7x+6 = 0 ⇒ (x-6)(x-1) =0 ⇒ x =6 or x =1
∴ present age of men = (7*6-6) = 36 yrs.

Q 30 - A mother was 4 times older in the comparison of her daughter before


the period of 10 year. The mother age would be double in the comparison of
her daughter after 10 year. What should be the present age of the daughter ?
A - 5 years
B - 10 years
C - 20 years
D - 30 years

Answer : C

Explanation

Let daughter age 10 year ago be x years.


Mother age 10 years ago = 4x years
2(x+20) = (4x+20) ⇒ 2x = 20 ⇒ x =10
Present age of daughter = (x+10) = 20 years.

Q 31 - Before the period of 16 year, my grandfather was 8 times older in the


comparison of me. He would be 3 times of my age 8 years now. Find out the
ratio of my grandfather and my age before the period of 8 year.
A - 1:2
B - 1:5
C - 3:8
D - none of these

Answer : D

Explanation

Let my age 16 yrs ago be x years.


My grandfather age at that time = 8x years
My present age = (x+16) yrs.
My grandfather present age = (8x+16) years.
3(x+16+8)= (8x+16+8) ⇒ 3x+72 = 8x+24 ⇒ 5x = 48 ⇒ x = 48/5
Ratio of our ages 8 yrs ago = (x+16-8) /(8x+16-8) = x+8/ 8x+8
(48/5+8)/ (8* 48/5 +5) = 88/424 = 11 :53.

Q 32 - Ajit age after 15 years will be 5 times his age 5 years back . what is the
present age of ajit ?
A - 5 years
B - 10 years
C - 15 years
D - 50 years

Answer : B

Explanation

Let the present age of ajit be x years.


(x+15) = 5(x-5) ⇒ x+15 = 5x-25 ⇒ 4x= 40 =. X = 10

Q 33 - If the ratio of ages of two persons Ram and sham is 5:4 . After Three
years their age ratio changes and becomes 11:9. In that case tell about the
present age of mr. sham.

A - 23 years

B - 24 years

C - 25 years

D - 26 years

Answer - B

Explanation

If the age of Mr. Ram 5x and 4x is the age of Mr. sham.


Then, ( 5x+3) /( 4x+3) = 11/9 ⇒ 9 (5x+3)= 11 (4x+3) ⇒ x= (33-27) = 6
So the present age of Mr. sham = 6*4 = 24 years.
Q 34 - A mother is 30 time older in the comparison of her daughter. After the
period of 18 year , the mother age would be thrice in the comparison of his
daughter . In that case tell about the present age of mother.

A - 40 years

B - 41 years

C - 42 years

D - 43 years

Answer - A

Explanation

Let daughter present age be x year .


in that case mother present age would be = 30x years
30x+ 18 = 3 (x+ 18) ⇒ 27x = 36 ⇒ x = 4/3
∴ so the present age of mother = (30* 4/3) = 40 years.

Q 35 - The ratio of present ages of three persons ajay , vijay and sanjay are in
the proportion of 4: 7: 9. Before 8 year total sum of their age is 56. What
should be the present ages?

A - 28 and 36 years

B - 28 and 38 years

C - 30 and 36 years
D - 36 and 28 years

Answer - A

Explanation

If the present age of ajay , vijay and sanjay is 4x , 7x and 9x years.


Total sum of ages of ajay, vijay and sanjay before 8 years ago = (4x-8)+(7x-
8)+(9x-8)
=(20x-24) years.
∴ 20x-24 = 56 ⇒ 20x = 80 ⇒ x= 4
Hence, it proves that age of ajay is 4*4 = 16 years,
vijay (7*4) = 28 years and sanjay (9* 4) = 36 years.

Q 36 - Daughter's present age is 2/5 in the comparison of her mother .8 year


later , age of her daughter will be 1/2 in the comparison of her mother. Find
out mother present age?

A - 39 years.

B - 40 years

C - 41 years

D - 42 years

Answer - B

Explanation

If the present age of mother is equal to x year.


In that situation the daughter present age would be = 2x/5 years.
2x/5 + 8 = 1/2 (x+8) ⇒ 4x+ 80 = 5x+40 ⇒ x = 40.
The mother age at the present time is = 40 years.

Q 37 - Ajay age was double in the comparison of bhuvan before 3 years. Seven
years hence, the sum of both ages would be 83 years. What should be the age
at the present time of both?

A - 43 years

B - 44 years

C - 45 years

D - 46 years

Answer - C

Explanation

Before 3 year let bhuvan age be x years.


3 years before , ajay age will be 2x years.
Now Bhuwan's age =(x+3) years and ajay age = (2x+3) years.
(x+3)+7+(2x+3)+7 = 83 ⇒ 3x+20 = 83 ⇒ 3x = 63 ⇒ x = 21
Now the bhuwan present age = (21+3) = 24 years
Now the ajay present age = (2 *21+3) years = 45 years.

Q 38 - I am 4 year older in the comparison of my sister, but my brother who is


the youngest among us is 7 year younger to myself. My father is three times in
the comparison of my brother. The present age of my sister 18 year and my
father is 3 year older in the comparison of my mother. In that situation what
should be the present age of my mother?.

A - 42 years
B - 43 years

C - 44 years

D - 45 years

Answer - A

Explanation

If my sister age is x years. Then,


Sister - x
I - x+4
Brother - (x+4-7) = x-3
Father - 3 (x-3)
Given x = 18
∴ Father's age = 3(18-3) = 45 years.
Mother age = (45-3) = 42 years.

Q 39 - Ajay is as much younger to vijay as he is older to vinay. If 48 years is the


sum of the ages of vijay and buwan . Then find out the present age of Mr. ajay
?

A - 21 years

B - 22 years

C - 23 years

D - 24 years

Answer - D

Explanation
V-A = A- B ⇒ V+B = 2A =48 ⇒ 24
Now, We can say that the present age of Mr. Ajay is 24 years.

Q 40 - If 100 year is equal to the sum of the ages of father and son. 2:1 was
the ratio of father and son before the period of 5 years. Find out the ratio of
ages which would be after the period 10 year.

A - 3:4

B - 3:5

C - 4:3

D - 5:3

Answer - D

Explanation

If the age of father at the present time = x years


His son age at the present time = (100-x) years.
x-5 / (100-x-5) = 2/1 ⇒ (x-5) = 2(95-x) ⇒ 3x = 195 ⇒ x = 65
Ratio of the ages of man and son after 10 years = (65+10)/(35+10)= 75/45 =
5/3 = 5:3

Q.41 - A father said his son , " I was as old as you are at present at the time of
your birth. " If the father age is 38 now, the son age 5 years back was :
A. 14
B. 19
C. 33
D. 38
Answer - A

Explanation

Let the son's present age be x years .Then, (38 - x) = x => x= 19.

Son's age 5 years back = (19 - 5) = 14 years

Q.42 - The total age of A and B is 12 years more than the total age of B and C. C
is how many years younger than A ?
A. 12
B. 13
C. 14
D. 15

Answer - A

Explanation

(A+B) - (B+C) = 12

A - C = 12.

C is younger than A by 12 years


Q 43 - The ages of Arzoo and Arnav are in the ratio of 11:13 respectively. After
7 years the ratio of their ages will be 20:23. What is the difference in years
between their ages?
A - 4 years
B - 7 years
C - 6 years
D - 5 years

Answer : C

Explanation

Let the present ages of Arzoo and Arnav be 11x and 13x respectively.
According to the question,
(11x + 7) / (13x + 7) = 20/23
Or, 26x + 140 = 253x + 161
Or, 260x - 253x = 161 - 140
Or, 7x = 21
Or, x = 3

Difference between their ages = 13x - 11x


= 2x
= 2*3
= 6 years

Q 44 - In a family the average age of the father and mother is 38 years


whereas the average age of father, mother and their only daughter is 28
years. Then the age of the daughter is -
A - 5 years
B - 6 years
C - 7 years
D - 8 years

Answer : D

Explanation

The age of the daughter = (3*28) - (2*38)


= 84 - 76
= 8 years

Q 45 - The ratio of the present ages of a son and his father is 1:5 and that of
his mother and father is 4: 5. After 2 years the ratio of the ages of the son to
that of his mother becomes 3 : 10. What is the present age of the father?
A - 37 years
B - 34 years
C - 30 years
D - 35 years

Answer : D

Explanation

Let the present age of the son be x years.


present age of the father = 5x years and that of the mother = 4x years.

2 years hence, age of son = (x + 2) years


And age of the will be = (4x + 2) years

As per the question,


(x + 2)/(4x + 2) = 3/10
Or, 10 (x+2) = 3 (4x + 2)
Or, 10x + 20 = 12x + 6
Or, 2x = 14
Or, x = 7
Present age of father = 5x = 5*7 = 35 years

Q 46 - A man five times older in the comparison of his son in the present time
but before the period of 5 year the father age in the comparison of his son
was 10 times older . Find out present age of the father?
A - 45 year
B - 48 years
C - 49 years
D - 52 years

Answer : A

Explanation

Let the present age of the son be x year. Then , present age of the man = 5x
year
∴ (5x-5 = 10 (x-5) ⇒ 5x= 45 ⇒ x = 9
Age at the present time of the father is = (5*9) = 45 year

Q 47 - If 100 year is equal to the sum of the ages of father and son. 2:1 was
the ratio of father and son before the period of 5 years . Find out the ratio of
ages which would be after the period 10 year.
A - 4:3
B - 5:3
C - 3:5
D - 10:7

Answer : B

Explanation

If x is the father age ,then son age will be (100-x)


x-5/ (100-x)-5 =2/1 ⇒ (x-5)= 2(95-x) ⇒ x =-5= 190 -2x ⇒ 3x =195 ⇒ x =65
father age = 65 yrs. And son age = 100-65 = 35 year
required ratio = (65+10) / (35+10) =75/45 = 5:3

Q 48 - What should be the present age of two person if 5:7 is the present age
ratio of two person and 3:5 was the age ratio before the period of 16 year?
A - 30, 44 years
B - 35, 52 years
C - 40, 56 years
D - 45, 60 years

Answer : C

Explanation

If the present age of two persons are 5x years and 7x years.


5x-16/ 7x-16 =3/5 ⇒ 5(5x-16) =3(7x-16)
⇒ 4x= (80-48) = 32 ⇒ x =8
Hence, their present ages are 40 years and 56 years.
Q 49 - A mother have 7 times in the comparison of her daughter at the time of
1 year ago . At the present time mother age is equal to the square of her son .
What should be the time age of mother?
A - 7 years
B - 36 years
C - 49 years
D - 64 years

Answer : B

Explanation

Let the present age of the son be x years.


Man present age =7(x-1)+1 years = (7x-6 ) yrs.
7x-6 = x2 ⇒ x2-7x+6 = 0 ⇒ (x-6)(x-1) =0 ⇒ x =6 or x =1
∴ present age of men = (7*6-6) = 36 yrs.

Q 50 - A mother was 4 times older in the comparison of her daughter before


the period of 10 year. The mother age would be double in the comparison of
her daughter after 10 year. What should be the present age of the daughter ?
A - 5 years
B - 10 years
C - 20 years
D - 30 years

Answer : C

Explanation
Let daughter age 10 year ago be x years.
Mother age 10 years ago = 4x years
2(x+20) = (4x+20) ⇒ 2x = 20 ⇒ x =10
Present age of daughter = (x+10) = 20 years.

Q 51 - Before the period of 16 year, my grandfather was 8 times older in the


comparison of me. He would be 3 times of my age 8 years now. Find out the
ratio of my grandfather and my age before the period of 8 year.
A - 1:2
B - 1:5
C - 3:8
D - none of these

Answer : D

Explanation

Let my age 16 yrs ago be x years.


My grandfather age at that time = 8x years
My present age = (x+16) yrs.
My grandfather present age = (8x+16) years.
3(x+16+8)= (8x+16+8) ⇒ 3x+72 = 8x+24 ⇒ 5x = 48 ⇒ x = 48/5
Ratio of our ages 8 yrs ago = (x+16-8) /(8x+16-8) = x+8/ 8x+8
(48/5+8)/ (8* 48/5 +5) = 88/424 = 11 :53.

Q 52 - Ajit age after 15 years will be 5 times his age 5 years back . what is the
present age of ajit ?
A - 5 years
B - 10 years
C - 15 years
D - 50 years

Answer : B

Explanation

Let the present age of ajit be x years.


(x+15) = 5(x-5) ⇒ x+15 = 5x-25 ⇒ 4x= 40 =. X = 10
TRUE DISCOUNT

Important Formulae
Let rate = R% per annum and Time = T years. Then,

 P.W. = (100 x Amount) / (100 + (R x T))

= (100 x T.D.) / (R x T)

 T.D. = (P.W. x R x T) / 100

= (Amount x R x T) / (100 + (R x T))

 Sum = (S.I. x T.D.) / (S.I. - T.D.)

 S.I. - T.D. = S.I. on T.D.

 When the sum is put at compound interest, then

P.W. = Amount / (1+R/100)T

Q1. Rama has to pay Rs 740 due in 4 years at 12 % per annum. Find the Principal
&true discount.

A.500 and 240

B.300 and 160

C. 200 and 180

D.100 and 80
Answer: A

Explanation:

P.W. =

= = Rs 500

T.D. = (Amount) – (P.W.) = ` (740 – 500) = Rs 240.

Q2. Due to covid situation Suresh’s electricity bill was due for 8 months hence at
10% per annum the true discount is Rs 480. Find the amount of the bill and its
present worth.

A.7680 and 7200

B.8460 and 7980

C. 6014 and 5534

D.5790 and 5310

Answer: A

Explanation:

= T.D ⇒ = 480

x = Rs 7680

Amount = Rs 7680

P.W. = (7680 - 480 ) = 7200

Q3. The simple interest on a certain sum of money for 4 years and at the certain
rate is Rs 425. The true discount on the same sum due for 4 years hence is Rs 340.
Find the the rate.
A.7 %

B.4 %

C.5 %

D.6 %

Answer: D

Explanation:

T.D. = Rs 340 and S.I. = Rs 425

∴ Sum due = = = 1700

Rate = = 6.25% = 6 %

Q4. The simple interest on a certain sum of money for 6 years and at the certain
rate is Rs 564. The true discount on the same sum due 6 year hence is Rs 420. Find
the sum and the rate percent.

A.7 %

B.4 %

C.5 %

D.6 %

Answer: C

Explanation:

T.D. = Rs 420 and S.I. = Rs 564


∴ Sum due = = = 1645

Rate = = 5.71% = 5 %

Q5. If Rs 20 be allowed as true discount on a bill of Rs 220 due at the end of a


certain period, then the discount allowed on the same sum due at the end of three
times the time is

A.50.76

B.30.24

C. 40.13

D.60

Answer: A

Explanation:

S.I. on Rs (220 – 20) for a certain time = Rs 20.

S.I. on Rs 200 for three times the time = 60.

T.D. on Rs 260 = Rs (260 – 200) = Rs 60.

T.D. on Rs 220 = 220 = Rs 50.76

Q6. Sham bought a smart lock for Rs 8000 and sold it the same day for Rs 9600,
allowing the buyer a credit of 2 years. If the rate of interest be 15% per annum, then
find outa profit/loss to Sham?

A.615.38 loss
B.832.34 profit

C. 729.10 loss

D.557.66 profit

Answer: A

Explanation:

C.P. = Rs 8000.

S.P. = = 7384.62

Loss = 8000 - 7384.62 = 615.38

Q7. Fruits were bought for Rs 800 and sold the same day for Rs 845.25 at a credit
of 8 months and thus gaining 4%. The rate of interest per annum is

A.8.48%

B.7.26%

C. 9.12%

D.10.22%

Answer: A

Explanation:

S.P = 104% of Rs 800 = 800 = Rs 832

Now, P.W. = Rs 800 and sum = Rs 845.25.

T.D. = Rs (845.25 – 800) = Rs 45.25.

Thus, S.I. Rs 800 for 8 months is Rs 45.25.


∴ Rate = = 8.48%

Q8. The present worth of Rs 2406 due in two equal half yearly installments at 12%
p.a. simple interest is

A.6123.01

B.3401.32

C. 4418.02

D.7246.21

Answer: C

Explanation:

Required sum = P.W. of Rs 2406 due 6 months hence + P.W. of Rs 2406 due 1 year
hence

= +

= Rs. 4418.02

Q9. Raju hasto pay Rs 960 due after 6 years hence at 18 % per annum. Find the true
discount.

A.461.53

B.325.31

C. 518.12

D.248.50
Answer: A

Explanation:

P.W. =

= = Rs 461.53

T.D. = (Amount) – (P.W.) = ` (960 – 461.5) = Rs 498.46

Q10. The simple interest on a certain sum of money for 5 years and at the certain
rate is Rs 625. The true discount on the same sum due 5 year hence is Rs 500. Find
the rate of true discount.

A.7%

B.4 %

C.5 %

D.6 %

Answer: B

Explanation:

T.D. = Rs 500 and S.I. = Rs 625

∴ Sum due = = = 2500

Rate = = 4%

Q11. The true discount on Rs 1840 due after a certain time at 15% p.a. is Rs 240. The
time after which it is due is

A.12 months
B.12 months

C. 14 months

D.10 months

Answer: A

Explanation:

P.W. = Rs (1840 – 240) = Rs 1600.

∴ S.I. on Rs 1600 at 15% = Rs 240

Time = = year

Time = 12 months

Q12. If Rs 40 is the true discount on Rs 340 due after a certain time. What will be the
true discount on the same sum due after half of the former time, the rate of interest
being the same?

A.46.41

B.62.33

C. 21.25

D.14.78

Answer: C

Explanation:

S.I. on Rs (340 – 40) for a given time = Rs 40.

S.I. on ` 300 for half the time = Rs 20.


T.D. on Rs 320 = Rs 20.

∴ T.D. on Rs 340 = = 21.25

Q13. Rahul purchased earbuds for Rs 1540 in cash and sold them for Rs1800 at a
credit of 6 months. If the rate of interest is 12% per annum, Rahul gains

A. 136.18

B. 162.73

C. 158.11

D.147.20

Answer: C

Explanation:

S.P. = P.W. of Rs 1800 due 6 months hence

S.P = = 1698.11

Gains = 1698.11 - 1540 =

= Rs 158.11

Q14. A Businessman owns a Food merchant Rs 30,500 due 2 year hence. The
Businessman wants to settle the account after 4 months. If the rate of interest is
12% per annum, how much cash should he pay?

A. 45634.12

B. 34213.34

C. 29326.92
D.14744.20

Answer: C

Explanation:

Required money = P.W. of Rs 30500 due 4 months hence

S.P =

= 29326.92

Q15. The present worth of Rs 1826 due in two equal yearly installments each at 15%
per annum simple interest is

A.5213.78

B.4021.61

C.3286.43

D.3415.34

Answer: C

Explanation:

Required sum = P.W. of Rs 1826 due 6 months hence + P.W. of Rs 1826 due 1 year
hence

= +

= Rs. 3286.43

Q16. Smartphone was bought for Rs 18000 and sold the same day for Rs. 19500.25
at a credit of 12 months and thus gaining 5%. The rate of interest per annum is

A.8.33%

B.7.26%
C. 9.12%

D.10.42%

Answer: A

Explanation:

S.P = 105% of Rs 18000 = 18000 = Rs 18900

Now, P.W. = Rs 18000 and sum = Rs 19500.25.

T.D. = Rs (19500.25 – 18000) = Rs 1500.25.

Thus, S.I. Rs 18000 for 12 months is Rs 1500.25.

∴ Rate = = 8.33%

Q17. Toys were bought for Rs 1000 and sold the same day for Rs 1224.25 at a credit
of 10 months and thus gaining 2%. The rate of interest per annum is

A.24.51%

B.32.17%

C. 51.83%

D.43.15%

Answer: A

Explanation:

S.P = 102% of Rs 1000 = 1000 = Rs 1020

Now, P.W. = Rs 1000 and sum = Rs 1224.25.

T.D. = Rs (1224.25 – 1020) = Rs 204.25.

Thus, S.I. Rs 1000 for 10 months is Rs 204.25.


∴ Rate = = 24.51%

Q18. A bill falls due in 2 years. The creditor agrees to accept immediate payment of
the two thirds and to defer the payment of the other one third for 3 years. By this
arrangement he gains Rs 80. What is the amount of the bill, if the money is worth
14 % ?

A.646.40

B.742.13

C. 577.29

D.802.86

Answer: A

Explanation:

Let the sum be ` x. Then,

⇒ - = 80

⇒( + )- = 80

⇒ x = 646.40

Q19. The product of simple interest and the true discount on a certain sum of money
for 8 months at 12 % p.a is 120. Find the sum.

A.7

B.15

C.8

D.12
Answer: D

Explanation:

Let the sum be ` x. Then,

T.D = =

S.I = x =

∴ = 120

⇒ x² = 18720

⇒ x = 12

Q20. The true discount on a certain sum of money due 6 years hence, is Rs 180 and
the simple interest on the same sum for the same time and at the same rate is Rs
220. Find rate of interest?

A.6 %

B.3 %

C.4 %

D.2 %

Answer: B

Explanation:

TD = Rs 180 and SI = Rs 220


Sum =

= Rs. 990

Rate = =3 %

Q21. The sum of simple interest and true discount on a certain sum of money for 8
months at 14 % p.a is 180. Find the sum.

A.955.05

B.846.23

C.986.18

D.969.34

Answer: A

Explanation:

Let the sum be Rs x. Then,

T.D = =

S.I = x =

∴ = 180

⇒ x = 955.05
Q22. An order was delivered for Rs 1200 and sold the same day for Rs 1500.15 at a
credit of 11 months and thus gaining 4%. The rate of interest per annum is

A.43.15%

B.32.17%

C. 51.83%

D.22.92%

Answer: D

Explanation:

S.P = 104% of Rs 1200 = 1200 = Rs 1248

Now, P.W. = Rs 1248 and sum = Rs 1500.15.

T.D. = Rs (1500.15 – 1248) = Rs 252.15.

Thus, S.I. Rs 1200 for 11 months is Rs 252.15.

∴ Rate = = 22.92%

Q23. The present worth of Rs 4720 due in two equal half yearly installments each at
12 % per annum simple interest is

A.6123.01

B.4401.32

C. 8647.62

D.7246.21

Answer: C
Explanation:

Required sum = P.W. of Rs 4720 due 6 months hence + P.W. of Rs 4720 due 1 year
hence

= +

= Rs. 8647.62

Q29. Sahil lent Rs 500 to Aditya for 2 years, and Rs 200 to Gopal for 4 years and
received together from both Rs 90 as interest. Find rate%, simple interest being
calculated?

A.15

B.5

C. 10

D.20

Answer: B

Explanation:

+ = 90

+ = 90

18R = 90

R =5%

Q24. Nilesh Purchased a set of cups at 1400 and sold it the same day for Rs 1600,
allowing the buyer a credit of 4 years. If the rate of interest be 15% per annum, then
the buyer has a profit of?
A.200

B.400

C. 300

D.600

Answer: B

Explanation:

C.P. = Rs 1400.

S.P. = = 1000

Gain to Buyer/Loss to Nilesh= 1400 - 1000 = 400

Q 25 - Find the P.W. of Rs. 9920 due 3 years at 8% p.a.


A - Rs 7000
B - Rs 8000
C - Rs 6000
D - Rs 7000

Answer - B
Explanation
P.W.= ((100 x Amount))/(100+(R x T))
= Rs {(100 x 9920)/(100 (3 x 8))}
= Rs (100 x 9920)/124
= Rs 8000

Q 26 - The genuine rebate on a bill due 9 months consequently at 6% for each


annum is Rs. 180. Discover its present worth.
A - Rs 4000
B - Rs 5000
C - Rs 3000
D - Rs 6000
Answer - A
Explanation
P.W. = ((100 x T.D.))/((R x T))
= Rs {(100 x 180)/(6 x 3/4)}
= Rs 4000

Q 27 - The genuine markdown on a sure total of cash due 3 years


consequently is Rs 200 and the straightforward enthusiasm on the some
aggregate for the same time and at the same guideline is Rs. 240. Discover the
aggregate and the rate percent.
A - Rs 1200
B - Rs 1000
C - Rs 1200
D - Rs 1000
Answer - A
Explanation
T.D = Rs 200 and S.I. = Rs 240
Sum due = (S .I. x T.D.)/(S.I.-T.D.)
= Rs ((240 x 200))/((240-200))
= Rs 1200

Q 28 - The genuine rebate on Rs 1860 due after a sure time at 5% p.a. is Rs.
60. Discover the time after which it is expected.
A - 10 months
B - 8 months
C - 7 months
D - 6 months

Answer - B
Explanation
P.W. = (Amount)-(T.D.)
= Rs (1860-60)
= Rs 1800
T.D. is S.I. on P.W.
Rs. 60 is S.I. on Rs 1800 at 5% p.a.
Time = ((100 x 60))/((5 x 1800)) years
= 2/3 years
= 2/3 x 12 months
= 8 months
Q 29 - Find the rebate on Rs. 9920 due 3 years at 8% p.a.
A - Rs 1720
B - Rs 1820
C - Rs 1920
D - Rs 1220

Answer - C
Explanation
P.W.= ((100 x Amount))/(100+(R x T))
= Rs {(100 x 9920)/(100 (3 x 8))}
= Rs (100 x 9920)/124
= Rs 8000
T.D. = (Amount)-(P.W.)
= Rs (9920-8000)
= Rs 1920

Q 30 - The genuine rebate on a bill due 9 months consequently at 6% for each


annum is Rs. 180. Discover the charge's measure.
A - Rs 4180
B - Rs 3180
C - Rs 2180
D - Rs 1180

Answer - A
Explanation
P.W. = ((100 x T.D.))/((R x T))
= Rs {(100 x 180)/(6 x 3/4)}
= Rs 4000
Sum = (P.W.+T.D.)
= Rs (4000+180)
= Rs 4180

Q 31 - The genuine markdown on a sure total of cash due 3 years


consequently is Rs 200 and the straightforward enthusiasm on the some
aggregate for the same time and at the same guideline is Rs. 240. Discover the
rate percent.
A - 50/3% p.a
B - 40/3% p.a
C - 10/3% p.a
D - 20/3% p.a

Answer - D
Explanation
T.D = Rs 200 and S.I. = Rs 240
Sum due = (S .I. x T.D.)/(S.I.-T.D.)
= Rs ((240 x 200))/((240-200))
= Rs 1200
T.D is S.I. on the sum.
Rs. 240 is S.I. on Rs 1200 for a long time.
R= ((100 x 240))/((1200 x 3))% p.a.
= 20/3% p.a
Q 32 - The genuine markdown on Rs 2575 due 4 months thus is Rs. 75.
Discover the rate of hobby.
A - 6% p.a.
B - 7% p.a.
C - 8% p.a.
D - 9% p.a.

Answer - D
Explanation
Amount = Rs 2575, T=4/12 years = 1/3 years, T.D = Rs. 75
P.W. = (Amount) - (T.D.) = Rs (2575-75) = 2500.
T.D. Is S.I. on P.W.
R.s 75 is S.I. on Rs. 2500 or 1/3 years
Rate = ((100x75)/(2500x1/3))% p.a. = 9% p.a.

Q 32 - The genuine rebate on a bill due 10 months consequently at 6% p.a. is


Rs 26.25. Discover the charge's measure.
A - Rs 451.25
B - Rs 551.25
C - Rs 351.25
D - Rs 251.25
Answer - B
Explanation
T.D. = Rs 26.25, T = 10/12 year= 5/6 year,R=6% p.a.
Let P.W. be Rs x. Then,
S.I. on Rs x at 6% p.a. for 5/6 year is Rs. 26.25
∴ (x * 6 * 5/6)/100 = 25.25
=> x= (26.25 * 20) = 525
∴ (P.W.) + (T.D.) = Rs. (525+26.25) = Rs 551.25

Q 33 - The contrast between the S.I. what's more, T.D. on a sure whole of cash
for 6 months at 6% p.a. is Rs. 27. Discover the total.
A - 32900
B - 30900
C - 31900
D - 33900

Answer - B
Explanation
S.I. = Rs. (x * 6 * 1/2)/100
= Rs. 3x/100 and T.D.
= Rs {(x * 78/12)/(100+(6 * 6/12) )}
= Rs. 3x/103
∴ 3x/100-3x/103 = 27
=> (309x-300x) = (27 * 100 * 103)
=> x = ((27*100*103))/9
= 30900
Q 34 - Find the single discount which is equivalent to successive discounts of
15% and 16%.
A - 36.6%
B - 31%
C - 28.6%
D - 29.4%

Answer : C

Explanation

Successive discount is calculated as X+Y-XY/100 %


Successive discount =15+16-15*16/100=31-240/100=28.6 %

Q 35 - Rs. 40 is the true discount on Rs. 520 due after a certain time. What will
be the true discount on the same sum due after half of the former time, the
rate of interest being the same?
A - Rs. 19.66
B - Rs. 20.66
C - Rs. 21.66
D - Rs. 22.66

Answer : C

Explanation

S.I. on Rs. (520 - 40) for a given time = Rs. 40.


S.I. on Rs. 480 for half the time = Rs. 20.
T.D. on Rs. 500 = Rs. 20
T.D. on Rs. 520 =Rs. ( 20/480 * 520)= Rs. 21.66

Q 36 - The present worth of a sum due sometime hence is Rs. 1024 and the
banker's gain is Rs.64. The true discount is:
A - Rs. 256
B - Rs. 125
C - Rs. 64
D - Rs. 32

Answer : A

Explanation

T.D. = √(P.W.x B.G.)


=√(1024x64.)
=Rs. 256

Q 37 - The banker's gain of a certain sum due 3 years hence at 20% per annum
is Rs. 48. The present worth is:
A - Rs.100
B - Rs.96
C - Rs.103.33
D - Rs.133.33

Answer : D

Explanation
T.D. =(B.G*100)/( R*T)=48*100/( 20*3)=Rs. 80.
Present worth=( 100*TD)/R*T=( 100*80)/20*3=Rs.133.33

Q 38 - The banker's discount on a sum of money for 2.5 years is Rs. 1125 and
the true discount on the same sum for 2 years is Rs.800. The rate percent is:
A - 6.25%
B-6%
C - 5.25%
D - 5%

Answer : A

Explanation

B.D. for 2.5(5/2) years = Rs. 1125.


B.D. for 2 years = Rs. (1125*( 2)/5*2) =Rs. 900.
T.D. for 2 years = Rs. 800.
Sum =( B.D*T.D )/(B.D*-T.D)=Rs.( ( 900*800 )/(900-800))=Rs. 900*800/100=Rs.
7200
Thus, Rs. 900 is SI on Rs. 7200 for 2 years
Rate = ( 100*900 )/7200*2=6.25%

Q 39 - The genuine markdown on an aggregate due 3 years consequently at


8% p.a. is Rs 252. The total due is:
A - Rs 798
B - Rs 1050
C - Rs 1302
D - None of these

Answer : C

Explanation

P.W. = (100*T.D.)/(R*T) = Rs ((100*252))/((8*3)) = Rs 1050


Sum due = (P.W.)+(T.D.) = Rs (1050+252) = Rs 1302

Q 40 - The straightforward hobby and the genuine rebate on a sure total due
6 months subsequently are Rs 25 and Rs 24 individually. The rate of interest
per annum is:
A - 8%
B - 17/2%
C - 35/4%
D - 25/3%

Answer : D

Explanation

((S.I.)*(T.D.))/((S.I.)-(T.D.))=Rs ((25*24))/((25-24))= Rs 600


S.I. on Rs 600 for 6 months is Rs 25
∴ Rate = (100*125)/(600*6/12)% p.a. = 25/3% p.a.

Q 41 - A man acquired a look for Rs 1950 in real money and sold it for Rs 2160
at a credit of 1 year. On the off chance that the rate of hobby is 8% p.a., then
the man has:
A - an increase of Rs 210
B - an addition of Rs 50
C - an addition of Rs 60
D - lost Rs 30

Answer : B

Explanation

C.P. = Rs. 1950


S.P. = P.W. of Rs. 2160 due 1 year hence
= Rs ((100*2160))/(100+(8*1)) = Rs 2000 in cash.
∴ Gain = Rs (2000-1950) = Rs 50

Q 42 - In the event that Rs 20 be permitted as genuine rebate on a bill of Rs


260 due after at some point, what will be the genuine markdown on the same
bill for half of the time?
A - Rs 10
B - Rs 10.40
C - Rs 13
D - Rs 15.20

Answer : B

Explanation

S.I. on Rs (260-20) for a fixed time = Rs 20


S.I. on Rs 240 for half the time = Rs 10
T.D. on Rs 250 = Rs 10
T.D. on Rs 260 = Rs (10/250*260) = Rs 10.40

Q 43 - A needs to pay Rs 1573 to B following 11/2 years and B needs to pay Rs


1444.50 to A following 6 months. In the event that the rate of hobby is 14%
for each annum and both need to settle the record at this time, who will pay
to the next and what amount?
A - A, Rs 28.50
B - B, Rs 37.50
C - A, Rs 50
D - B, Rs 50

Answer : D

Explanation

P.W. of loan on A = Rs ((100*1573))/(100+(14*2/3) )= Rs ((100*1573))/121= Rs


1300
P.W. of loan on B = Rs ((100*1444.50))/(100+(14*1/2) )= Rs ((144450))/103= Rs
1350
So, B will pay Rs. 50
PROFIT AND LOSS
1. Cost Price, (c.p.) = The price, at which an article is purchased, is called
its cost price.
2. Selling price (s.p) = The price, at which an article is sold, is called its
selling price.
3. Profit or Gain = (S.P) - (C.P)
4. Loss = (C.P) - (S.P)
5. Gain or Loss is always reckoned on C.P.

Formulae

1. Gain% = (Gain*100) / C.P


2. Loss% = (Loss*100) / C.P
3. S.P = (100+ Gain %) /100 * (C.P)
4. S.P = (100-Loss %) / 100 * (C.P)
5. C.P = 100 / (100 + Gain %)* (S.P)
6. C.P = 100 / (100-Loss %)* (S.P)

Important cases

1. If an article is sold at a profit of say, 20%, then SP = 120% of CP.


2. If an article is sold at a loss of say, 20%, then SP = 80% of CP.
3. When a person sells two similar items, one at a gain of say x% and the
other at a loss of say x%. then the seller always incurs a loss given by:
Loss% = (x/10)2

4. If a seller sells his goods at cost price but uses false weights, then

Gain% = [Error/(True value - Error) * 100]%

Q1. Sahil purchased a gaming laptop for Rs. 1,10,000 and sold it for Rs 1,48,000. What
is the percent profit he made on the laptop?

A. 44.13%

B. 27.22%

C. 34.54%

D. 56.71%

Answer: C

Explanation:

C.P. = Rs 1,10,000 ; S.P. = Rs 1,48,000.

Profit = Rs (148000 – 110000) = Rs 38000.

Profit % = % = 34.54%

Q2. Niraj bought a diamond ring for Rs. 4,50,000 and sold it for Rs 6,48,000. What is
the percent profit he made on the diamond ring?

A. 44%

B. 22%
C. 30.55%

D. 20%

Answer: A

Explanation:

C.P. = Rs 450000 ; S.P. = Rs 648000.

Profit = Rs (648000 – 450000) = Rs 198000.

Profit % = % = 44%

Q3. Find S.P., when C.P. = Rs 46.26, profit = 25% ?

A. 57.83

B. 46.24

C. 22.66

D. 11.56

Answer: A

Explanation:

C.P. = Rs 46.26 ; profit. = Rs 25%.

S.P. = 125% of Rs 46.26 = = Rs 57.83


Q4. Silver watch is sold for Rs 74645 at a loss of 12%. What is the cost price of the
silver watch?

A.Rs 72640.62

B.Rs 78926.23

C.Rs 84823.86

D.Rs 86123.71

Answer: C

Explanation:

loss = 12% ; S.P. = Rs 74645.

Profit = Rs = Rs 84823.86

Q5. Bunty sold an iphone at the cost of Rs 45600 at a loss of 20%. At what cost will
he have to sell it to get a gain of 25%?

A.Rs 56890

B.Rs 64540

C.Rs 71250

D.Rs 87360
Answer: C

Explanation:

Let the new S.P. be ` x. Then

(100 – loss%) : (1st S.P.) = (100 + gain%) : (2nd S.P.)

⇒ (100 – 20) : 45600 = (100 + 25) : x

⇒ 80 : 45600 = 125 : x

⇒x = = 71250

Q6. An automobile manufacturer earns 35% profit by selling each car for Rs 452000.
If the production cost is increased by 20%, what should be the new selling price of a
car so as to get a profit of 20%?

A.Rs 334814.8

B.Rs 526423.8

C. Rs 482133.6

D.Rs 401778.0

Answer: C

Explanation:

S.P. = Rs 452000, Profit = 35%


C.P. = = Rs 334814.81

New S.P. = = Rs 401778

Required S.P. = = Rs 482133.6

Q7. An Autobiography was sold for Rs 140.50 with a profit of 12%. If it were sold for
Rs 135.75, then what would have been the percentage of gain or loss?

A.6.17%

B.5.63%

C. 9.36%

D.8.21%

Answer: D

Explanation:

S.P. = Rs 140.50, Profit = 12%.

So, C.P. = = Rs 125.44

When S.P. = Rs 135.75, profit = Rs (135.75 – 125.44) = Rs 10.31.

Profit % = = 8.21%
Q8. A manufacturer makes 646 products at a cost of Rs 3.5 per product. He fixes
the selling price such that if only 426 products are sold, he would make a profit of
25% on his outlay. However, he sold 510 products at this price. Find his actual profit
percent of the total outlay, assuming that the unsold articles are useless.

A.42.3%

B.47.6%

C.38.9%

D.49.7%

Answer: D

Explanation:

C.P. of 646 products = Rs (3.50 × 646) = Rs 2261.

S.P. of 426 products = = Rs 2826.25

S.P. of 510 products = = Rs 3383.53

Profit = Rs (3383.53 – 2261) = Rs 1122.53

Profit % = = 49.7%
Q9. A vendor purchased 8 dragon fruits at Rs.160 and sold 6 of them at Rs. 100. Find
his gain or loss percent.

A.15.34%

B.16.67%

C. 83.33%

D.14.78%

Answer: B

Explanation:

Suppose, number of dragon fruit bought = L.C.M. of 8 and 6 = 24.

C.P.= = Rs 480

S.P. = = Rs 400

Loss % = = 16.67%

Q10. A seller bought 3 pencils at a rupee. How many for a rupee must he sell to gain
26%?

A.1.26

B.2.38

C.3.26
D.5.29

Answer: B

Explanation:

C.P. of 3 pencils = Rs 1, Gain = 26%. S.P. of 3 pencils == = Rs 1.26

For Rs 1.26 pencils sold = 3. For Rs 1, pencils sold = = 2.38

Q11. Ram bought 21 liter of oil at Rs 90.50 per liter and mixed it with 100 liter oil at Rs
115 per liter. At what rate should he sell the mixture to gain 12%?

A.135.22

B.124.03

C. 115.63

D.136.12

Answer: B

Explanation:

C.P. of 121 kg of mixture = Rs (21 × 90.50 + 100 × 115) = Rs 13400

S.P. = 112% of Rs 13400 = = Rs 15008


Rate of S.P. of the mixture = ) = 124.03 per liter

Q12. A vegetable dealer professes to sell vegetables at cost price but uses a fake
weight and earns a profit of 20%. What is a false weight instead of 1 kilogram?

A.144.44 gms

B.154.78 gms

C.177.57 gms

D.166.67 gms

Answer: D

Explanation:

Let the weight substitute for 1 kg be x

Gain% = x 100

20 = x 100

= 166.67 gms

Q13. If the producer gains 12%, arvind as a middle man 16% and rahul as a retailer
20%, then what is the cost of making a cupboard, the retail price of which is Rs
5640?
A.1964.24

B.2963.33

C.4608.20

D.3617.61

Answer: D

Explanation:

Let the cost of production of the table be ` x.

Then, 112% of 116% of 120% of x = 5640

⇒ x x x = 5640

⇒ = 3617.61

Q14. Shahrukh bought a car at 7/8 of its selling price and sold it at 6% more than its
S.P. Find his gain percent?

A.23.23%

B.15.35%

C.18.21%

D.21.14%

Answer: D
Explanation:

Let the S.P. be Rs x.

Then, C.P. =

Receipt = 106% of Rs x =

Gain = - =

Gain% = x x 100 = 21.14%

Q15. The price of a golden bracelet, passing through four persons, increases on the
total by 85%. If the 1st person, 2nd person and third person earned 15%, 20% and 25%
profit respectively, find the percentage gain by the fourth person.

A.7.25%

B.8.22%

C.12.50%

D.25%

Answer: A

Explanation:

Let the original price of the bracelet be Rs R

let the profit earned by the fourth seller be x%


Then, (100 + x)% of 115% of 120% of 125% of P = 185% of P

x x x x =

⇒ = 107.24 - 100 = 7.25%

Q16. A broker sold two row houses for Rs 8,65,424 each. On the first row house he
gains 18% while on the other he loses 18%. How much does he gain or lose in the
whole deal?

A.3.24

B.2.22

C.4.32

D.1.05

Answer: A

Explanation:

In such a case, there is always a loss. The selling price is immaterial.

Loss% = ( ) ^2 = (18/10)^2 = 3.24%


Q17. Gopal bought a monitor and CPU for Rs 24000. He sold the monitor at a gain of
18% and the CPU at a loss of 10%, thereby gaining 2% on the whole. Find the cost of
the monitor.

A.10285.7

B.11840.3

C.7218.4

D.13600.2

Answer: A

Explanation:

Let the C.P. of the monitor be Rs x.

Then, C.P. of the CPU= (24000 - x)

18% of x – 10% of (24000 – x) = 2% of 24000

⇒ - = 480

⇒ = 10285.7

Q18. An ill-informed salesperson labeled all the products at 40% above the cost
price and thinking that he can still make 24% profit, offers a discount of 24% on the
labeled price. What is the actual gain on the sales?

A.6.40%
B.8.20%

C.7.10%

D.5.90%

Answer: A

Explanation:

Let C.P. = Rs 100. Then, marked price = Rs 140

S.P. = 76% of Rs 140 = Rs 106.4.

∴ Gain% = 6.40%.

Q19. In the supermarket a person is selling jowar at a gain of 12% and he was using
a weighing machine that was weighing 24% less than the actual weight. The total
profit gained by the person is ?

A.65.56%

B.36.78%

C.47.36%

D.55.65%

Answer: C

Explanation:
Gain% = (( x 100) %

⇒ (( x 100) %

⇒ (( x 100) % = 47.36%

Q20. Sahil sold his old bike in 150000 and made two parts of the money he got and
invested in two different companies. In one he got 12% gain and in another 14%. If the
gain% are interchanged with the investment he would have got Rs.140 less. Find the
ratio between his investments in the two companies.

A.216 : 229

B.157 : 143

C.143 : 157

D.229 : 216

Answer: C

Explanation:

Let x be his investment in the first company and Y be in the second company.

We have,

⇒ 1.12X + 1.14Y = 1.14X + 1.12Y + 140


⇒ 0.02Y = 0.02X + 140

⇒ 2Y = 2X + 14000

⇒ Y = X + 7000

Since X + Y = 150000

2X = 143000

⇒ X = 71500

⇒ Y = 78500

So, X : Y = 143 : 157

Q21. Radhe and Shyam general stores mark the price of an item similar. Radhe
gives 2 back to back discounts of 20% each. Shyam store give a 20% discount on the
price and a subsequent discount of 29%. Which of the following is correct?

A.The price of the article is cheaper at Radhe store

B.The price of the article is cheaper at Shyam store

C.The price of the article is the same at Radhe and Shyam.

D.The price cannot be determined.

Answer: B

Explanation:

Let the M.P. of the item at each of the stores Radhe and Shyam be Rs 100
Final price at Radhe = 80% of 80% of 100

=( x x ) = Rs 64

Final price at Shyam = 71% of 80% of Rs 100

=( x x ) = Rs 56.8

Q22. Raju purchased a cricket kit and paid 15% less than its actual price. He sold it
and gained 30% on the price he had paid. The percentage gained by Raju on the
actual price was?

A.12.5

B.8.5

C.10.5

D.14.5

Answer: C

Explanation:

Let the original price of the cricket kit be Rs 100

⇒ Discounted price = Rs 85. Profit = 30%.

⇒ S.P. = 130% of Rs 85 = x ) = 110.5


So, profit percentage on actual price is 10.5%

Q23.Akshay sold an article for Rs 146 and got a loss. Had he sold the article for Rs
204, his profit would have been equal to one third of the amount of loss that he got.
At what price should he sell the article to have 18% gain?

A.196.69

B.346.22

C.274.94

D.432.73

Answer: C

Explanation:

Let the cost price of an article be x.

Then, loss = (146 – x)

Again, profit = (204 – x)

According to given information we get

Now, (204 – x) =

X =233

If profit % = 18% then


∴ Selling price = = 274.94

Q24. The profit earned by selling a Sofa for Rs 24000 is 1.4 times the loss incurred
when the same sofa was sold for Rs 20000. What is the cost of the sofa?

A.45888.88

B.16333.33

C.21666.66

D.52222.22

Answer: C

Explanation:

Let C.P. of sofa be Rs x

According to the question,

24000 – x = (x – 20000) × 1.4

⇒ 24000 - x = 1.4x - 28000

⇒ x = 21666.66

Q25. The price of Indian smartphones rose by 16% and later on reduced by 18% due
to less sales. Find the total percentage change in the final price of the smartphone?
A.4.6%

B.4.2%

C.4.9%

D.4.4%

Answer: C

Explanation:

Let the price of a smartphone be Rs 100.

First increase% = 16%

After increase, price of smartphone

=100 x

Second decrease% = 18%

After decrease, price of article = 116 - = Rs 95.12

Net change percent in price = x100 = 4.9%

Q26. By selling 26 meters of steel cable, one gains the selling price of 12 meter. Find
the profit percent

A.66.92%

B.72.44%
C. 92.63%

D.85.71%

Answer: D

Explanation:

(S.P. of 26 m) – (C.P. of 26 m) = Gain = S.P. of 12 m.

S.P. of 14 m = C.P. of 26 m.

Let C.P. of each meter be Rs 1.

Then, C.P. of 14 m = Rs 14 , S.P. of 14 m = Rs 26.

Profit % = = 85.71%

Q 27 - A vendor bought 6 oranges for Re 10 and sold them at 4 for Re 6. Find


his loss or gain percent.
A - 8% gain
B - 10% gain
C - 8% loss
D - 10% loss

Answer - D
Explanation
Suppose, number of oranges bought = LCM of 6 and 4 = 12
∴CP = Re (10/6 * 12) = Re 20 and SP = Re (6/4 * 12) = Re 18
∴Loss% = (2/20 * 100)% = 10%
Q 28 - By selling 33 meters of cloth, one gains the selling price of 11 meters.
Find the gain percent.
A - 50%
B - 45%
C - 40%
D - 60%

Answer - A
Explanation
(SP of 33m) - (CP of 33m) = Gain = SP of 11m
∴ SP of 22m = CP of 33m
Let CP of each meter be Re 1. Then, CP of 22m = Re 22.
Hence SP of 22m = Re 33.
∴ %Gain = 11/22 * 100
= 50%

Q 29 - Pure ghee costs Re 100 per kg. A shopkeeper mixes vegetable oil
costing Re 50 per kg and sells the mixture at Re 96 per kg, making a profit of
20%. In what ratio does he mix the pure ghee with the vegetable oil.
A - 3:2
B - 2:3
C - 4:3
D - 3:1

Answer - A
Explanation
Mean Cost price = Re (100/120)*96 = Re 80 per kg
Apply rule of allegation,
there4; Required ratio = 30:20 = 3:2

Q 30 - The CP of 25 articles is equal to SP of 20 articles. Find the loss or gain


percent.
A - 35%
B - 30%
C - 25%
D - None of these

Answer - C
Explanation
Let the CP of each article = Re 1.
Then CP of 20 articles = Re 20.
SP of 20 articles = CP of 25 articles = Re 25.
∴ Gain% = (5/20)*100% = 25%

Q 31 - A shopkeeper bought 80 kg of sugar at Re 13.50/kg and mixed it with


120 kg sugar at Re 16/kg. If he is to make a profit of 16% what rate should he
sell the sugar to his customers?
A - Re 12/kg
B - Re 15.25/kg
C - Re 17/kg
D - Re 17.40/kg

Answer - D
Explanation
CP of 200 kg of mixture = Re (80 * 13.50) + (120 * 16) = Re 3000
SP = 116% of Re 3000 = Re (116/100)*3000 = Re 3480
∴Rate of SP = Re 3480/200 = Re 17.40/kg

Q 32 - A man bought cookies at 3 for a rupee. How many for a rupee should
he sell to make a profit a 50%.
A-1
B-2
C - 1.5
D - None of these

Answer - B
Explanation
CP of 3 cookies = Re 1
SP of 3 cookies = 150% of Re 1 = 3/2
For Re 3/2, the man sells 3 cookies.
Hence for Re 1, number of cookies sold = 3*2/3 = 2

Q 33 - Anil buys a calculator for Re 600 and sells it to Vikash at 10% profit.
Vikash sells it to Chandan for 5 % profit. Chandan after using it for certain
time, sells it to Dinesh at a loss of 20%. For how much Chandan sell the
calculator to Dinesh.
A - Re 550.50
B - Re 564.40
C - Re 554.40
D - None of these

Answer - C
Explanation
SP for Chandan = 600 * (110/100) * (105/100) * (80/100)
= 600 * 924/1000
= Re 554.40

Q 34 - An article is sold by X to Y at a loss of 20%, Y to Z at a gain of 15%, Z to


W at a loss of 5% and W to V at a profit of 10%. If v had to pay Re 500, how
much X paid for it?
A - Re 520.07
B - Re 490.07
C - Re 510.07
D - Re 530.07

Answer - A
Explanation
CP for X = 500 * (100/80) * (100/115) * (100/95) * (100/110)
= 500 * 10000/9614
= Re 520.07

Q 35 - A vendor when could not find buyers for his vegetable at Re 10/kg,
reduced the rate to Re 8.10 per kg but uses a faulty weight of 900 gm in place
of 1 kg weight. Find the percent change in the actual price or loss.
A - 8%
B - 8.10%
C - 9%
D - 10%
Answer - D
Explanation
After the price was reduced, 900 gm now costs Re 8.10.
Hence 1000gm will cost (1000/900)*8.10 = Re 9
% change in actual price or loss = [(10 - 9)/10]*100%
= 10%

Q 36 - A trader marks the SP of an object at a profit of 20%. Considering the


demand o the object, he further increases the price by 10%. Find the final
profit %.
A - 35%
B - 31%
C - 32%
D - 25%

Answer - C
Explanation
Let the CP = Re 100
∴ SP = 100 * (120/100) * (110/100)
= Re 132

Final profit = (132 - 100)*100%


= 32%

Q 37 - An article when sold for Re 4600 makes a 15% profit. Find the profit or
loss % if it was sold for Re 3600.
A - 10% gain
B - 11% loss
C - 10% loss
D - 11% gain

Answer - C
Explanation
CP = 4600 * (100/115)
=Re 4000
Loss% = [(4000-3600)/4000]*100%
= 10%

Q 38 - A seller sells a watch at 5% loss. If he had bought it at 20% more and


sold it for Re 115 less, he would have incurred a loss of 40%. Find the cost
price of the watch.
A - Re 500
B - Re 5000
C - Re 550
D - Re 450

Answer - A
Explanation
Assume CP = x
Selling price at the first case = (95/100)x
Selling price at the second case = (60/100)*(120/100)x
= (7200/10000)x

As per question,
(95/100)x - (7200/10000)x = 150
Or, x = Re 500
Q 39 - When a man sold an article for Re 540, he made a loss of 10%. At what
price should he sell it, so that he incurs a loss of only 5%.
A - Re 550
B - Re 525
C - Re 575
D - Re 570

Answer - D
Explanation
CP = 540*(100/90)
= Re 600

New SP = 600*(95/100)
= Re 570

Q 40 - Ram sells chocolates at a profit of 20% for Re 60. What will be the
percentage loss or gain if he reduces the price to Re 55 due to less demand.
A - 11%
B - -11%
C - 10%
D - -10%

Answer - C
Explanation
CP = 60*(100/120)
= Re 50
New SP = Re 55
Gain% = (5/50)*100
=10%

Q 41 - A shopkeeper buys rice for Re 1600. He had to sell 1/4th at a loss of


20%. If he is to make an overall gain of 10%, what percentage of profit he
needs to make out of the remaining stock of rice?
A - 20%
B - 25%
C - 15%
D - 18%

Answer - A
Explanation
CP of 1/4th of the stock = 1600/4 = Re 400
SP of 1/4th of the stock = 400*(80/100)
=Re 320

In order to make a profit of 10% on total CP, the SP should be:


SP = 1600*(110/100)
= Re 1760

∴The SP for the remaining 3/4th of the stock


should be Re 1760 - Re 320 = Re 1440.

Cost Price of the 3/4th of stock


= Re 1600 - Re 400 = Re 1200.
∴%Gain = [(1440-1200)/1200*100}]
= (240/1200)*100
= 20%
Q 42 - A 10% hike in the price of wheat forces a person to purchase 2 kg less
for Re 110. Find the new and the original price of the wheat.
A - Re 10/kg
B - Re 5/kg
C - Re 6/kg
D - Re 8/kg

Answer - B
Explanation
10% of Re 110 = Re 11
Cost of 2 kg of wheat at new price = Re 11
So, cost of 1 kg of wheat at new price = Re 5.50 = Re 11/2

Original Price = (11/2)*(100/110)


= Re 5 per kg

Q 43 - 10 kg of rice costs as much as 20 kg of wheat, 25 kg of wheat costs as


much as 2kg of tea, 5 kg of tea costs as much as 25kg of sugar. Find the cost of
6 kg of sugar if 14 kg of rice costs Re 32.
A - Re 50
B - Re 55
C - Re 60
D - Re 65
Answer - C
Explanation
4 kg of rice costs Re 32
∴10kg of rice will cost = (32/4)*10 = Re 80
20 kg of wheat costs Re 80.
∴25kg of wheat costs = (80/20)*25 = Re 100

2kg of tea costs Re 100


∴5 kg of tea costs = (100/2)*5 = Re 250

25kg of sugar costs Re 250.


∴6 kg of sugar costs = (250/25)*6 = Re 60

Q 44 - A fruit seller sells bananas at a profit of 20%. If he increases the selling


price of each banana by 25 paisa, he earns a profit of 45%. Find the initial
selling price of each banana and also its cost price.
A - SP = Re1.20, CP = Re 1
B - SP = Re1.50, CP = Re 1
C - SP = Re1.20, CP = Re 1.10
D - None of the above.

Answer - A
Explanation
Let CP = x paisa.
Initial SP = x*(120/100) paisa

As per question,
120x/100 + 25 = (145/100)x
or, 145x/100 - 120x/100 = 25
or, 25x/100 = 25
or, x = 100 paisa

CP = 100 paisa or Re 1.
Initial SP = 120 paisa or Re 1.20.
Q 45 - A man sold two plots for Re 8 lakhs each. One on he earns a profit of
16% and the other he loses 16%. How much does he loss or gain in the whole
transaction?
A - 2.5% loss
B - 3% gain
C - 2.56% loss
D - 3.56% loss

Answer - C
Explanation
Applying direct formula, %loss = (16/10)2%
= 64/25%
= 2.56%

Q 46 - An uneducated retailer marks all his goods at 50% above the cost price
and thinking that he will still make 25% profit, offers a discount of 25% on the
marked price. What is his actual profit on the sales?
A - 10%
B - 12.50%
C - 11.50%
D - 12%

Answer - B
Explanation
Let CP = Re 100.
The, marked price, MP = Re 150
SP = 75% of Re 150 = Re 112.50
∴ Gain% = 12.50%

Q 47 - A book shop offers a book at an increase of 10%. On the off chance that
he had purchased it at 4% less and sold it for Rs. 6 more, he would have
picked up 75/4 %. The expense cost of the book is:
A - 130
B - 140
C - 150
D - 160

Answer : C

Explanation

Let the C.P. be Rs x.Then, S.P. =Rs (110/100 * x) =Rs 11x/10.


New C.P. = 96% of Rs x=Rs (96/100 * x) =Rs 24x/25.
New S.P. =Rs (11x/10+6).
∴ (11x/10+6) =475/4% of 24x/25⇒11x+60/10=475/400*24x/25=57x/50
⇒550x+3000=570x
⇒20x=3000⇒x=150.
∴ C.P. =Rs 150.

Q 48 - Mohan purchased 20 feasting tables for rs. 12000 and sold them at a
benefit equivalent to the offering cost of 4 eating tables. The offering cost of
every eating table is:
A - 700
B - 750
C - 725
D - 775
Answer : B

Explanation

C.P of each table = rs. (12000/20) = rs. 600


(S.P of 20 tables)- (C.P of 20% table) = profit = S.P of 4 tables
⇒S.P of 16 tables = C.P of 20 tables= 1200
⇒ S.P of 1 table = (12000/16) = Rs. 750

Q 49 - A man offers an article at 12.5% misfortune. Had he sold it for Rs.


103.60 more, he could have picked up 6%. What is the C.P of the articles?
A - 278.60
B - 350
C - 432
D - 560

Answer : D

Explanation

Let the C.P be Rs. x. Then ,


(106/100 *x) - (87.5/100*x) =103.60
⇒ 106x- 87.5x= 103.60
⇒ 18.5x =10360, x= 103600/185 = 560
∴ C.P= Rs. 560

Q 49 - A shipper has 1000kg of sugar, a portion of which he offers at 8%


benefit what's more, and the rest at 18% benefit. He increases 14% in general.
The amount sold at 18% benefit is:
A - 560 kg
B - 600 kg
C - 400 kg
D - 640 kg

Answer : B

Explanation

Let the sugar sold at *5 gain be x kg.


Then, sugar sold at 18% gain = (1000-x) kg
Let the C.P Of sugar be Rs. Y per Kg.
Total C.P = Rs.(1000y)
(108/100*xy)+ 118/100 (1000-x)y = 114/100 *1000y
⇒108xy + 118000y -118xy = 114000y
⇒ 10x =4000 ⇒ x= 400
Quantity sold at 18% gain= (1000-400) = 600 kg

Q 50 - The profit earned after selling an article for Re 625 is the same as the
loss incurred after selling the article for Re 435. What is the cost price of the
article?
A - Re 530
B - Re 520
C - Re 550
D - None of these

Answer : A

Explanation

Let profit = loss = Re x

As per question,
625 ? x = 435 + x
Or, 2x = 190
Or, x = Re 95

∴ CP = Re (435 + 95) = Re 530

Q 51 - A shopkeeper expects a gain of 17.5% on his cost price. If in a week, his


sale was of Rs. 235, what was his profit?
A - 35
B - 32.5
C - 30
D - 27.5

Answer : A

Explanation

C.P. = Rs 100/( 117.5) x 235=Rs.200


Profit = 235-200=Rs. 35

Q 52 - The aggregate expense cost of two watches is R. 840. One is sold at a


benefit of 16% and the other at lost 12%. There is no misfortune or addition in
the entire exchange. The expense cost of the watch on which the
businessperson additions, is:
A - 360
B - 370
C - 380
D - 390

Answer : A
Explanation

Let their C.P be rs. x and Rs. (840-x) resp. Then,


(116/100*x) + 88/100*(840-x) ⇒ 29x/25+ (22 (840-x)/25) = 840
⇒ (29x-22x) + 18480 =21000 ⇒ 7x =2520
∴ Required C.P = Rs. 360

Q 53 - Which of the accompanying is the most reduced proportion?


A - 7:13
B - 17:25
C - 7:15
D - 15:23

Answer : C

Explanation

7/13 =0.538, 17/25 =0.68, 7/15= 0.466, 15/23 =0.652


Clearly, 7/15 is the smallest.

Q 54 - A businessperson denotes his merchandise to gain35%. In any case, he


permits 10% markdown for money installment. His benefit percent is:
A - 27/2%
B - 43/2%
C - 25%
D - 63/2%

Answer : B

Explanation
Let the C.P be Rs.100. Then, M.p= rs. 135
S.P = 90% of rs. 135= Rs. (90/100*135)= rs. 121.50
Profit% = (121.50-100)% =21.50% =43/2%

Q 55 - On a Rs. 10000 installment arrange a man has decision between three


progressive rebates of 10%, 10% and 30% and three progressive rebates of
40%, 5% and 5%. By picking the better one, he can spare:
A - Rs. 200
B - Rs. 255
C - Rs. 400
D - Rs. 433

Answer : B

Explanation

1st payment= 90% 0f 90% of 70% of rs. 10000


= rs. (90/100*90/100* 70/100*10000) =5670
2nd. Payment = 60% of 95% of 95% of 10000.
= (60/100*95/100*95/100*10000)= 5415
By choosing the better one he can save rs . (5670-5415) = 255

You might also like